Mental Health Exam 3

Pataasin ang iyong marka sa homework at exams ngayon gamit ang Quizwiz!

Which behavior best demonstrates aggression? a. Stomping away from the nurses' station, going to the hallway, and grabbing a tray from the meal cart. b. Bursting into tears, leaving the community meeting, and sitting on a bed hugging a pillow and sobbing. c. Telling the primary nurse, "I felt angry when you said I could not have a second helping at lunch." d. Telling the medication nurse, "I am not going to take that, or any other, medication you try to give me."

a Aggression is harsh physical or verbal action that reflects rage, hostility, and potential for physical or verbal destructiveness. Aggressive behavior violates the rights of others. Refusing medication is a client's right and may be appropriate. The other incorrect options do not feature violation of another's rights.

Two staff nurses applied for promotion to nurse manager. The nurse not promoted initially had feelings of loss but then became supportive of the new manager by helping make the transition smooth and encouraging others. Which term best describes the nurse's response? a. Altruism b. Suppression c. Intellectualization d. Reaction formation

a Altruism is the mechanism by which an individual deals with emotional conflict by meeting the needs of others and receiving gratification vicariously or from the responses of others. The nurse's reaction is conscious rather than unconscious. There is no evidence of suppression. Intellectualization is a process in which events are analyzed based on remote, cold facts and without passion, rather than incorporating feeling and emotion into the processing. Reaction formation is when unacceptable feelings or behaviors are controlled and kept out of awareness by developing the opposite behavior or emotion.

What is the priority nursing intervention when caring for a client after an overdose of amphetamines? a. Monitor vital signs. b. Observe for depression. c. Awaken the client every 15 minutes. d. Use warmers to maintain body temperature.

a An overdose of stimulants, such as amphetamines, can produce respiratory and circulatory dysfunction as well as hyperthermia. Concentration is impaired. This client will be hypervigilant; it is not necessary to awaken the client.

Which is an effective nursing intervention to assist an angry client learn to manage anger without violence? a. Help a client identify a thought that produces anger, evaluate the validity of the belief, and substitute reality-based thinking. b. Provide negative reinforcement such as restraint or seclusion in response to angry outbursts, whether or not violence is present. c. Use aversive conditioning, such as popping a rubber band on the wrist, to help extinguish angry feelings. d. Administer an antipsychotic or antianxiety medication.

a Anger has a strong cognitive component, so using cognition techniques to manage anger is logical. The incorrect options do nothing to help the client learn anger management.

The nurse who is counseling a client with dissociative identity disorder should understand that which assessment is of the highest priority? a. risk for self-harm. b. cognitive function. c. memory impairment. d. condition of self-esteem.

a Assessments that relate to client safety take priority. Clients with dissociative disorders may be at risk for suicide or self-mutilation, so the nurse must be alert for indicators of risk for self-injury. The other options are important assessments but rank below safety. Treatment motivation, while an important consideration, is not necessarily a part of the nursing assessment.

Which features should be present in a therapeutic milieu for a client experiencing a hallucinogen overdose? a. Simple and safe b. Active and bright c. Stimulating and colorful d. Confrontational and challenging

a Because the individual who has ingested a hallucinogen is probably experiencing feelings of unreality and altered sensory perceptions, the best environment is one that does not add to the stimulation. A simple, safe environment is a better choice than an environment with any of the characteristics listed in the other options. The other options would contribute to a "bad trip."

Which comment by a client who recently experienced a myocardial infarction indicates use of maladaptive, ineffective coping strategies? a. "My employer should have paid for a health club membership for me." b. "My family will see me through this. It won't be easy, but I will never be alone." c. "My heart attack was no fun, but it showed me up the importance of a good diet and more exercise." d. "I accept that I have heart disease. Now I need to decide if I will be able to continue my work daily."

a Blaming someone else and rationalizing one's failure to exercise are not adaptive coping strategies. Seeing the glass as half full, using social and religious supports, and confronting one's situation are seen as more effective strategies. The distracters demonstrate effective coping associated with a serious medical condition.

A nurse assesses a client diagnosed with conversion (functional neurological) disorder. Which comment is most characteristic of this client? a. "Since my father died, I've been short of breath and had sharp pains that go down my left arm, but I think it's just indigestion." b. "I have daily problems with nausea, vomiting, and diarrhea. My skin is very dry, and I think I'm getting seriously dehydrated." c. "Sexual intercourse is painful. I pretend as if I'm asleep so I can avoid it. I think it's starting to cause problems with my marriage." d. "I get choked very easily and have trouble swallowing when I eat. I think I might have cancer of the esophagus."

a Clients with conversion (functional neurological) disorder demonstrate a lack of concern regarding the seriousness of symptoms. This lack of concern is termed la belle indifférence. There is also a specific, identifiable cause for the development of the symptoms; in this instance, the death of a parent would precipitate stress. The distracters relate to sexual dysfunction and illness anxiety disorder.

At a meeting for family members of alcoholics, a spouse says, "I did everything I could to help. I even requested sick leave when my partner was too drunk to go to work." The nurse assesses these comments as what? a. codependence. b. assertiveness. c. role reversal. d. homeostasis.

a Codependence refers to participating in behaviors that maintain the addiction or allow it to continue without holding the user accountable for his or her actions. The other options are not supported by information given in the scenario.

Which assessment findings support a diagnosis of ODD? a. Negative, hostile, and spiteful toward parents. Blames others for misbehavior. b. Exhibits involuntary facial twitching and blinking; makes barking sounds. c. Violates others' rights; cruelty toward people or animals; steals; truancy. d. Displays poor academic performance and reports frequent nightmares.

a ODD is a repeated and persistent pattern of having an angry and irritable mood in conjunction with demonstrating defiant and vindictive behavior. The distracters identify findings associated with CD, anxiety disorder, and Tourette's syndrome.

During the third week of treatment, the spouse of a client in a rehabilitation program for substance abuse says, "After this treatment program, I think everything will be all right." Which remark by the nurse will be most helpful to the spouse? a. "While sobriety solves some problems, new ones may emerge as one adjusts to living without drugs and alcohol." b. "It will be important for you to structure life to avoid as much stress as you can and provide social protection." c. "Addiction is a lifelong disease of self-destruction. You will need to observe your spouse's behavior carefully." d. "It is good that you are supportive of your spouse's sobriety and want to help maintain it."

a During recovery, clients identify and use alternative coping mechanisms to reduce reliance on substances. Physical adaptations must occur. Emotional responses were previously dulled by alcohol but are now fully experienced and may cause considerable anxiety. These changes inevitably have an effect on the spouse and children, who need anticipatory guidance and accurate information.

An adolescent was arrested for prostitution and assault on a parent. The adolescent says, ―I hate my parents. They focus all attention on my brother, who's perfect in their eyes.‖ Which type of therapy might promote the greatest change in the adolescent's behavior? a. Family therapy b. Bibliotherapy c. Play therapy d. Art therapy

a Family therapy focuses on problematic family relationships and interactions. The patient has identified problems within the family. Play therapy is more appropriate for younger patients. Art therapy and bibliotherapy would not focus specifically on the identified problem.

Parents of an adolescent diagnosed with a CD say, ―We don't know how to respond when our child breaks the rules in our house. Is there any treatment that might help us?‖ Which therapy is likely to be helpful for these parents? a. Parent-child interaction therapy (PCIT) b. Behavior modification therapy c. Multi-systemic therapy (MST) d. Pharmacotherapy

a In PCIT, the therapist sits behind one-way mirrors and coaches parents through an ear audio device while they interact with their children. The therapist can suggest strategies that reinforce positive behavior in the adolescent. The goal is to improve parenting strategies and thereby reduce problematic behavior. Behavior modification therapy may help the adolescent, but the parents are seeking help for themselves. MST is much broader and does not target the parents' need.

How do relaxation techniques help clients who have experienced major traumas? a. By engaging the parasympathetic nervous system. b. By increasing sympathetic stimulation. c. By increasing their metabolic rate. d. By releasing hormones.

a In response to trauma, the sympathetic arousal symptoms of rapid heart rate and rapid respiration prepare the person for flight or fight responses. Afterward, the dorsal vagal response damps down the sympathetic nervous system. This is a parasympathetic response with the heart rate and respiration slowing down and decreasing the blood pressure. Relaxation techniques promote activity of the parasympathetic nervous system.

A client in the emergency department shows disorganized behavior and incoherence after a friend suggested a homosexual encounter. In which room should the nurse place the client? a. An interview room furnished with a desk and two chairs b. A small, empty storage room with no windows or furniture c. A room with an examining table, instrument cabinets, desk, and chair d. The nurse's office, furnished with chairs, files, magazines, and bookcases

a Individuals experiencing severe to panic-level anxiety require a safe environment that is quiet, non-stimulating, structured, and simple. A room with a desk and two chairs provides simplicity, few objects with which the client could cause self-harm, and a small floor space in which the client can move about. A small, empty storage room without windows or furniture would feel like a jail cell. The nurse's office or a room with an examining table and instrument cabinets may be over-stimulating and unsafe.

The staff development coordinator plans to teach use of physical management techniques for use when clients become assaultive. Which topic should the coordinator emphasize? a. Practice and teamwork b. Spontaneity and surprise c. Caution and superior size d. Diversion and physical outlets

a Intervention techniques are learned behaviors and must be practiced to be used in a smooth, organized fashion. Every member of the intervention team should be assigned a specific task to carry out before beginning the intervention. The other options are useless if the staff does not know how to use physical techniques and how to apply them in an organized fashion.

A client who was responding to auditory hallucinations earlier in the morning now approaches the nurse shaking a fist and shouts, "Back off!" and then goes to the dayroom. While following the client into the dayroom, the nurse should take what precaution? a. make sure there is adequate physical space between the nurse and client. b. move into a position that places the client close to the door. c. maintain one arm's length distance from the client. d. begin talking to the client about appropriate behavior.

a Making sure space is present between the nurse and the client avoids invading the client's personal space. Personal space needs increase when a client feels anxious and threatened. Allowing the client to block the nurse's exit from the room may result in injury to the nurse. Closeness may be threatening to the client and provoke aggression. Sitting is inadvisable until further assessment suggests the client's aggression is abating. One arm's length is inadequate space.

An adolescent diagnosed with CD has aggression, impulsivity, hyperactivity, and mood symptoms. The treatment team believes this adolescent may benefit from medication. The nurse anticipates the health care provider will prescribe which type of medication? a. Second- generation antipsychotic b. Antianxiety medication c. Calcium channel blocker d. β-blocker

a Medications for CD are directed at problematic behaviors such as aggression, impulsivity, hyperactivity, and mood symptoms. Second-generation antipsychotics are likely to be prescribed. β-blocking medications may help to calm individuals with intermittent explosive disorder by slowing the heart rate and reducing blood pressure. Calcium channel blockers reduce blood pressure but are not used for persons with impulse control problems. An antianxiety medication will not assist with impulse control.

A client admitted to an alcohol rehabilitation program tells the nurse, "I'm actually just a social drinker. I usually have a drink at lunch, two in the afternoon, wine with dinner, and a few drinks during the evening." The client is using which defense mechanism? a. Denial b. Projection c. Introjection d. Rationalization

a Minimizing one's drinking is a form of denial of alcoholism. The Client is more than a social drinker. Projection involves blaming another for one's faults or problems. Rationalization involves making excuses. Introjection involves incorporating a quality of another person or group into one's own personality.

An individual experiences sexual dysfunction and blames it on a partner by calling the person unattractive and unromantic. Which defense mechanism is evident? a. Rationalization b. Compensation c. Introjection d. Regression

a Rationalization involves unconsciously making excuses for one's behavior, inadequacies, or feelings. Regression involves the unconscious use of a behavior from an earlier stage of emotional development. Compensation involves making up for deficits in one area by excelling in another area. Introjection is an unconscious, intense identification with another

A person speaking about a rival for a significant other's affection says in an emotional, syrupy voice, "What a lovely person. That's someone I simply adore." The individual is demonstrating which defense mechanism? a. reaction formation. b. repression. c. projection. d. denial.

a Reaction formation is an unconscious mechanism that keeps unacceptable feelings out of awareness by using the opposite behavior. Instead of expressing hatred for the other person, the individual gives praise. Denial operates unconsciously to allow an anxiety-producing idea, feeling, or situation to be ignored. Projection involves unconsciously disowning an unacceptable idea, feeling, or behavior by attributing it to another. Repression involves unconsciously placing an idea, feeling, or event out of awareness.

A nurse assesses an individual who commonly experiences anxiety. Which comment by this person indicates the possibility of obsessive-compulsive disorder? a. "I check where my car keys are eight times." b. "My legs often feel weak and spastic." c. "I'm embarrassed to go out in public." d. "I keep reliving a car accident."

a Recurring doubt (obsessive thinking) and the need to check (compulsive behavior) suggest obsessive-compulsive disorder. The repetitive behavior is designed to decrease anxiety but fails and must be repeated. Stating "My legs feel weak most of the time" is more in keeping with a somatic disorder. Being embarrassed to go out in public is associated with an avoidant personality disorder. Reliving a traumatic event is associated with posttraumatic stress disorder.

A client fearfully runs from chair to chair crying, "They're coming! They're coming!" The client does not follow the staff's directions or respond to verbal interventions. What is the initial nursing intervention of highest priority? a. providing for the client's safety. b. encouraging clarification of feelings. c. respecting the client's personal space. d. offering an outlet for the client's energy.

a Safety is of highest priority because the client experiencing panic is at high risk for self-injury related to increased non-goal-directed motor activity, distorted perceptions, and disordered thoughts. Offering an outlet for the client's energy can occur when the current panic level subsides. Respecting the client's personal space is a lower priority than safety. Clarification of feelings cannot take place until the level of anxiety is lowered.

Which assessment question could a nurse ask to help identify secondary gains associated with a somatic symptom disorder? a. "What are you unable to do now but were previously able to do?" b. "How many doctors have you seen in the last year?" c. "Who do you talk to when you're upset?" d. "Did you experience abuse as a child?"

a Secondary gains should be assessed. Secondary gains reinforce maladaptive behavior. The client's dependency needs may be evident through losses of abilities. When secondary gains are prominent, the client is more resistant to giving up the symptom. There may be a history of abuse or doctor shopping, but the question does not assess the associated gains.

A soldier in a combat zone tells the nurse, "I saw a child get blown up over a year ago, and I still keep seeing bits of flesh everywhere. I see something red, and the visions race back to my mind." Which phenomenon associated with post-traumatic stress disorder (PTSD) is the soldier describing? a. Re-experiencing b. Hyperarousal c. Avoidance d. Psychosis

a Spontaneous or cued recurrent, involuntary, and intrusive distressing memories of the traumatic events are often associated with PTSD. The soldier has described intrusive thoughts and visions associated with reexperiencing the traumatic event. This description does not indicate psychosis, hypervigilance, or avoidance.

What is the most therapeutic characteristics for a nurse working with a client beginning treatment for alcohol addiction to present? a. Empathetic, supportive b. Skeptical, guarded c. Cool, distant d. Confrontational

a Support and empathy assist the client to feel safe enough to start looking at problems. Counseling during the early stage of treatment needs to be direct, open, and honest. The other approaches will increase client anxiety and cause the client to cling to defenses.

A student says, "Before taking a test, I feel very alert and a little restless." Which nursing intervention is most appropriate to assist the student? a. Explain that the symptoms result from mild anxiety and discuss the helpful aspects. b. Advise the student to discuss this experience with a health care provider. c. Encourage the student to begin antioxidant vitamin supplements. d. Listen attentively, using silence in a therapeutic way.

a Teaching about symptoms of anxiety, their relation to precipitating stressors, and, in this case, the positive effects of anxiety will serve to reassure the client. Advising the client to discuss the experience with a health care provider implies that the client has a serious problem. Listening without comment will do no harm but deprives the client of health teaching. Antioxidant vitamin supplements are not useful in this scenario.

A nurse wants to research epidemiology, assessment techniques, and best practices regarding persons with addictions. Which resource will provide the most comprehensive information? a. Substance Abuse and Mental Health Services Administration (SAMHSA) b. Institute of Medicine (IOM)-National Research Council c. National Council of State Boards of Nursing (NCSBN) d. American Society of Addictions Medicine

a The SAMHSA is the official resource for comprehensive information regarding addictions. The other resources have relevant information, but they are not as comprehensive.

When a client first began using alcohol, two drinks produced relaxation and drowsiness. After 1 year, four drinks are needed to achieve the same response. Why has this change occurred? a. Tolerance has developed. b. Antagonistic effects are evident. c. Metabolism of the alcohol is now delayed. d. Pharmacokinetics of the alcohol have changed.

a Tolerance refers to needing higher and higher doses of a drug to produce the desired effect. The potency of the alcohol is stable. Neither hypomagnesemia nor antagonistic effects account for this change.

1. A 16-year-old diagnosed with a conduct disorder (CD) has been in a residential program for 3 months. Which outcome should occur before discharge? a. The adolescent and parents create and agree to a behavioral contract with rules, rewards, and consequences. b. The adolescent identifies friends in the home community who are a positive influence. c. Temporary placement is arranged with a foster family until the parents complete a parenting skills class. d. The adolescent experiences no anger and frustration for 1 week.

a The adolescent and the parents must agree on a behavioral contract that clearly outlines rules, expected behaviors, and consequences for misbehavior. It must also include rewards for following the rules. The adolescent will continue to experience anger and frustration. The adolescent and parents must continue with family therapy to work on boundary and communication issues. It is not necessary to separate the adolescent from the family to work on these issues. Separation is detrimental to the healing process. While it is helpful for the adolescent to identify peers who are a positive influence, it is more important for behavior to be managed for an adolescent diagnosed with a CD.

Which scenario demonstrates a dissociative fugue? a. After being caught in an extramarital affair, a man disappeared but then reappeared months later with no memory of what occurred while he was missing. b. A man is extremely anxious about his problems and sometimes experiences dazed periods of several minutes passing without conscious awareness of them. c. A woman finds unfamiliar clothes in her closet, is recognized when she goes to new restaurants, and complains of "blackouts" despite not drinking. d. A woman reports that when she feels tired or stressed, it seems like her body is not real and is somehow growing smaller.

a The client in a dissociative fugue state relocates and lacks recall of his life before the fugue began. Often fugue states follow traumatic experiences and sometimes involve assuming a new identity. Such persons at some point find themselves in their new surroundings, unable to recall who they are or how they got there. A feeling of detachment from one's body or from the external reality is an indication of depersonalization disorder. Losing track of several minutes when highly anxious is not an indication of a dissociative disorder and is common in states of elevated anxiety. Finding evidence of having bought clothes or gone to restaurants without any explanation for these is suggestive of dissociative identity disorder, particularly when periods are "lost" to the client (blackouts).

Which assessment made by a client best supports dissociative fugue? a. "I cannot recall why I'm living in this town." b. "I feel as if I'm living in a fuzzy dream state." c. "I feel like different parts of my body are at war." d. "I feel very anxious and worried about my problems."

a The client in a fugue state frequently relocates and assumes a new identity while not recalling previous identity or places previously inhabited. The distracters are more consistent with depersonalization disorder, generalized anxiety disorder, or dissociative identity disorder.

What is the priority outcome for a client completing the fourth alcohol detoxification program in the past year? a. Prior to discharge, the client will state, "I know I need long-term treatment." b. Prior to discharge, the client will use denial and rationalization in healthy ways. c. Prior to discharge, the client will identify constructive outlets for expression of anger. d. Prior to discharge, the client will develop a trusting relationship with one staff member.

a The correct response recognizes the need for ongoing treatment after detoxification and is the best goal related to controlling relapse. The scenario does not give enough information to determine whether anger has been identified as a problem. A trusting relationship, while desirable, should have occurred earlier in treatment.

A wife received news that her husband died of heart failure and called her family to come to the hospital. She angrily tells the nurse who cared for him, "He would still be alive if you had given him your undivided attention." What is the nurse's best intervention? a. Say to the wife, "I understand you are feeling upset. I will stay with you until your family comes." b. Say to the wife, "Your husband's heart was so severely damaged that it could no longer pump." c. Say to the wife, "I will call the health care provider to discuss this matter with you." d. Hold the wife's hand in silence until the family arrives.

a The nurse builds trust and shows compassion in the face of adjustment disorders. Therapeutic responses provide comfort. The nurse should show patience and tact while offering sympathy and warmth. The distracters are defensive, evasive, or placating.

After major reconstructive surgery, a client's wounds dehisced. Extensive wound care was required for 6 months, causing the client to miss work and social activities. Which physiological response would be expected for this client? a. Vital signs return to normal. b. Release of endogenous opioids would cease. c. Pulse and blood pressure readings are elevated. d. Psychomotor abilities of the right brain become limited.

a The scenario presents chronic and potentially debilitating stress. The helpless and out of control feelings produce pathophysiological changes. Unmyelinated ventral vagus responses initially result in rapid heart rate and respiration. After many hours, days, or months the body cannot sustain this state, so the dorsal vagal response dampens the sympathetic nervous system. This parasympathetic response results in the heart rate and respiration slowing down and a decrease in blood pressure. Individuals with dissociative disorders have altered communication between higher and lower brain structures due to the massive release of endogenous opioids at the time of severe threat.

A client who experienced a myocardial infarction was transferred from critical care to a step- down unit. The client then used the call bell every 15 minutes for minor requests and complaints. Staff nurses reported feeling inadequate and unable to satisfy the client's needs. When the nurse manager intervenes directly with this client, which comment is most therapeutic? a. "I'm wondering if you are feeling anxious about your illness and being left alone." b. "The staff are concerned that you are not satisfied with the care you are receiving." c. "Let's talk about why you use your call light so frequently. It is a problem." d. "You frustrate the staff by calling them so often. Why are you doing that?"

a This client is experiencing anxiety associated with a serious medical condition. Verbalization is an effective outlet for anxiety. "I'm wondering if you are anxious ..." focuses on the emotions underlying the behavior rather than the behavior itself. This opening conveys the nurse's willingness to listen to the client's feelings and an understanding of the commonly seen concern about not having a nurse always nearby as in the intensive care unit. The other options focus on the behavior or its impact on nursing and do not help the client with her emotional needs.

Which assessment findings suggest the possibility of a factitious disorder, imposed on self- type? (Select all that apply.) a. History of multiple hospitalizations without findings of physical illness b. History of multiple medical procedures or exploratory surgeries c. Going from one doctor to another seeking the desired response d. Claims illness to obtain financial benefit or other incentive e. Difficulty describing symptoms

a, b Persons with factitious disorders, imposed on self-type, typically have a history of multiple hospitalizations and medical workups, with negative findings from workups. Sometimes they have even had multiple surgeries seeking the origin of the physical complaints. If they do not receive the desired response from a hospitalization, they may elope or accuse staff of incompetence. Such persons usually seek treatment through a consistent health care provider rather than doctor shopping, are not motivated by financial gain or other external incentives, and present symptoms in a very detailed, plausible manner indicating considerable understanding of the disorder or presentation they are mimicking. See relationship to audience response question.

A 10-year-old child was placed in a foster home after being removed from parental contact because of abuse. The child has apprehension, tremulousness, and impaired concentration. The foster parent also reports the child has an upset stomach, urinates frequently, and does not understand what has happened. What helpful measures should the nurse suggest to the foster parents? (Select all that apply.) a. conveying empathy and acknowledging the child's distress. b. explaining and reinforcing reality to avoid distortions. c. using a calm manner and low, comforting voice. d. avoiding repetition in what is said to the child. e. staying with the child until the anxiety decreases. f. maximizing opportunities for exercise and play.

a, b, c, e, f The child's symptoms and behavior suggest that he is exhibiting PTSD. Interventions appropriate for this level of anxiety include using a calm, reassuring tone, acknowledging the child's distress, repeating content as needed when there is impaired cognitive processing and memory, providing opportunities for comforting and normalizing play and physical activities, correcting any distortion of reality, and staying with the child to increase his sense of security.

The nurse interviewing a client with suspected post-traumatic stress disorder (PTSD) should be alert to which client findings? (Select all that apply.) a. avoids people and places that arouse painful memories. b. experiences flashbacks or re-experiences the trauma. c. experiences symptoms suggestive of a heart attack. d. feels compelled to repeat selected ritualistic behaviors. e. demonstrates hypervigilance or distrusts others. f. feels detached, estranged, or empty inside.

a, b, c, e, f These assessment findings are consistent with the symptoms of PTSD. Ritualistic behaviors are expected in obsessive-compulsive disorder.

The nurse assesses an adult who is socially withdrawn and hoards. Which nursing diagnoses most likely apply to this individual? (Select all that apply.) a. Ineffective home maintenance b. Situational low self-esteem c. Chronic low self-esteem d. Disturbed body image e. Risk for injury

a, c, e Shame regarding the appearance of one's home is associated with hoarding. The behavior is usually associated with chronic low self-esteem. Hoarding results in problems of home maintenance, which may precipitate injury. The self-concept may be affected, but not body

A young adult says, "I was sexually abused by my older brother. During those assaults, I went somewhere else in my mind. I don't remember the details. Now, I often feel numb or unreal in romantic relationships, so I just avoid them." Which disorders should the nurse suspect based on this history? (Select all that apply.) a. Acute stress disorder b. Depersonalization disorder c. Generalized anxiety disorder d. Post-traumatic stress disorder (PTSD) e. Reactive attachment disorder f. Disinhibited social engagement disorder

a, b, d Acute stress disorder, depersonalization disorder, and PTSD can involve dissociative elements, such as numbing, feeling unreal, and being amnesic for traumatic events. All three disorders are also responses to acute stress or trauma, which has occurred here. The distracters are disorders not evident in this client's presentation. Generalized anxiety disorder involves extensive worrying that is disproportionate to the stressors or foci of the worrying. Reactive attachment disorder and disinhibited social engagement disorder are problems of childhood.

A nurse assesses a client suspected of having somatic symptom disorder. Which assessment findings regarding this client support the suspected diagnosis? (Select all that apply.) a. Female b. Reports frequent syncope c. Rates pain as "1" on a scale of "10" d. First diagnosed with a skin rash at age 12 e. Reports insomnia often resulting from back pain

a, b, d, e Common symptoms for primary care visits are chest pain, fatigue, dizziness, headache, swelling, back pain, shortness of breath, insomnia, abdominal pain, paralysis, unexplained skin rashes and numbness. This disorder is more common in women than in men. Clients with conversion disorder would have a tendency to underrate pain.

A child placed in a foster home after being removed from abusive parents is apprehensive and overreacts to environmental stimuli. The foster parents ask the nurse how to help the child. Which interventions should the nurse suggest? (Select all that apply.) a. Use a calm manner and low voice. b. Maintain simplicity in the environment. c. Avoid repetition in what is said to the child. d. Minimize opportunities for exercise and play. e. Explain and reinforce reality to avoid distortions.

a, b, e The child has moderate anxiety. A calm manner will calm the child. A simple, structured, predictable environment is desirable to decrease anxiety provoking and reduce stimuli. Calm, simple explanations that reinforce reality validate the environment. Repetition is often needed when the individual is unable to concentrate because of elevated levels of anxiety. Opportunities for play and exercise should be provided as avenues to reduce anxiety. Physical movement helps channel and lower anxiety. Play helps by allowing the child to act out concerns.

Because an intervention was required to control a client's aggressive behavior, the nurse plans a critical incident debriefing with staff members. Which topics should be the primary focus of this discussion? (Select all that apply.) a. Client behaviors associated with the incident b. Genetic factors associated with aggression c. Intervention techniques used by the staff d. Effects of environmental factors e. Theories of aggression

a, c, d The client's behavior, the intervention techniques used, and the environment in which the incident occurred are important to establish realistic outcomes and effective nursing interventions. Discussing views about the theoretical origins of aggression would be less effective and relevant.

A nurse plans health teaching for a client diagnosed with generalized anxiety disorder who begins a new prescription for lorazepam. What information should be included? (Select all that apply.) a. Caution in use of machinery b. Foods allowed on a tyramine-free diet c. The importance of caffeine restriction d. Avoidance of alcohol and other sedatives e. Take the medication on an empty stomach

a, c, d Caffeine is a central nervous system stimulant that acts as an antagonist to the benzodiazepine lorazepam. Daily caffeine intake should be reduced to the amount contained in one cup of coffee. Benzodiazepines are sedatives, thus the importance of exercising caution when driving or using machinery and the importance of not using other central nervous system depressants such as alcohol or sedatives to avoid potentiation. Benzodiazepines do not require a special diet. Food will reduce gastric irritation from the medication.

A nurse's neighbor says, "I saw a news story about a man without any known illness who died suddenly after his ex-wife committed suicide. Was that a coincidence, or can emotional shock be fatal?" The nurse should respond by noting that what serious medical conditions may be complicated by emotional stress? (Select all that apply.) a. cancer. b. hip fractures. c. hypertension. d. immune disorders. e. cardiovascular disease.

a, c, d, e A number of diseases can be worsened or brought to awareness by intense emotional stress. Immune disorders can be complicated associated with detrimental effects of stress on the immune system. Others can be brought about indirectly, such as cardiovascular disease due to acute or chronic hypertension. Hip fractures are not in this group.

A nurse directs the intervention team who places an aggressive client in seclusion. Before approaching the client, which actions will the nurse direct team members to take? (Select all that apply.) a. Appoint a person to clear a path and open, close, or lock doors. b. Quickly approach the client and take the closest extremity. c. Select the person who will communicate with the client. d. Move behind the client when the client is not looking. e. Remove jewelry, glasses, and harmful items.

a, c, e Injury to staff and the client should be prevented. Only one person should explain what will happen and direct the client. This may be the nurse or a staff member with a good relationship with the client. A clear pathway is essential because those restraining a limb cannot use keys, move furniture, or open doors. The nurse is usually responsible for administering medication once the client is restrained. Each staff member should have an assigned limb rather than just grabbing the closest. This system could leave one or two limbs unrestrained. Approaching in full view of the client reduces suspicion.

The nurse can assist a client to prevent substance abuse relapse by (Select all that apply.) a. rehearsing techniques to handle anticipated stressful situations. b. advising the client to accept residential treatment if relapse occurs. c. assisting the client to identify life skills needed for effective coping. d. advising isolating self from significant others until sobriety is established. e. informing the client of physical changes to expect as the body adapts tofunctioning without substances.

a, c, e Nurses can be helpful as a client assesses needed life skills and in providing appropriate referrals. Anticipatory problem solving and role playing are good ways of rehearsing effective strategies for handling stressful situations and helping the client evaluate the usefulness of new strategies. The nurse can provide valuable information about physiological changes expected and ways to cope with these changes. Residential treatment is not usually necessary after relapse. Clients need the support of friends and family to establish and maintain sobriety.

After discovering discrepancies and missing controlled substances, the nursing supervisor determines that a valued, experienced staff nurse is responsible. Which actions should the nursing supervisor take? (Select all that apply.) a. Refer the nurse to a peer assistance program. b. Confront the nurse in the presence of a witness. c. Immediately terminate the nurse's employment. d. Relieve the nurse of responsibilities for client care. e. Require the nurse to undergo immediate drug testing.

a, d Registered nurses may have personal substance use problems. The nursing supervisor should provide for safe client care by relieving the nurse of responsibility for client care. For those nurses experiencing addictions, there are nonpunitive alternatives to discipline programs in the form of peer assistance. Many state boards of nursing have developed an alternative to discipline program to help impaired nurses. Terminating the nurse's employment and confronting the nurse in the presence of a witness are punitive actions. The peer assistance program will manage drug testing.

A nurse on an adolescent psychiatric unit assesses a newly admitted 14-year-old. An impulse control disorder is suspected. Which aspects of the patient's history support the suspected diagnosis? (Select all that apply.) a. Family history of mental illness b. Allergies to multiple antibiotics c. Long history of severe facial acne d. Father with history of alcohol abuse e. History of an abusive relationship with one parent

a, d, e Parents who are abusive, rejecting, or overly controlling cause a child to suffer detrimental effects. Other stressors associated with impulse control disorders can include major disruptions such as placement in foster care, severe marital discord, or a separation of parents. Substance abuse by a parent is common. Acne and allergies are not aspects of the history that relate to the behavior.

A client with a history of command hallucinations approaches the nurse yelling obscenities. Which nursing actions are most likely to be effective in de-escalation for this scenario? (Select all that apply.) a. Stating the expectation that the client will stay in control. b. Asking the client, "Do you want to go into seclusion?" c. Telling the client, "You are behaving inappropriately." d. Offering to provide the client with medication to help. e. Speaking in a firm but calm voice.

a, d, e Stating the expectation that the client will maintain control of behavior reinforces positive, healthy behavior and avoids challenging the client. Offering as-needed medication provides support for the client trying to maintain control. A firm but calm voice will likely comfort and calm the client. Belittling remarks may lead to aggression. Criticism will probably prompt the client to begin shouting.

Which central nervous system structures are most associated with anger and aggression? (Select all that apply.) a. Amygdala b. Cerebellum c. Basal ganglia d. Temporal lobe e. Prefrontal cortex

a, d, e The amygdala and prefrontal cortex mediate anger experiences and help a person judge an event as either rewarding or aversive. The temporal lobe, which is part of the limbic system, also plays a role in aggressive behavior. The basal ganglia are involved in movement. The cerebellum manages equilibrium, muscle tone, and movement.

A client diagnosed with a somatic symptom disorder says, "Why has God chosen me to be sick all the time and unable to provide for my family? The burden on my family is worse than the pain I bear." Which nursing diagnoses apply to this client? (Select all that apply.) a. Spiritual distress b. Decisional conflict c. Adult failure to thrive d. Impaired social interaction e. Ineffective role performance

a, e The client's verbalization is consistent with spiritual distress. The client's description of being unable to provide for and burdening the family indicates ineffective role performance. No data support diagnoses of adult failure to thrive, impaired social interaction, or decisional conflict.

A client fearfully runs from chair to chair crying, "They're coming! They're coming!" The client does not follow the staff's directions or respond to verbal interventions. Which nursing diagnosis has the highest priority? a. Fear b. Risk for injury c. Self-care deficit d. Disturbed thought processes

b A client experiencing panic-level anxiety is at high risk for injury related to increased non- goal-directed motor activity, distorted perceptions, and disordered thoughts. Data are not present to support a nursing diagnosis of self-care deficit or disturbed thought processes. The client may be afraid, but the risk for injury has a higher priority since it involves the potential of physical danger.

A nurse wants to teach alternative coping strategies to a client experiencing severe anxiety. Which action should the nurse perform first? a. Verify the client's learning style. b. Lower the client's current anxiety. c. Create outcomes and a teaching plan. d. Assess how the client uses defense mechanisms.

b A client experiencing severe anxiety has a markedly narrowed perceptual field and difficulty attending to events in the environment. A client experiencing severe anxiety will not learn readily. Determining preferred modes of learning, devising outcomes, and constructing teaching plans are relevant to the task but are not the priority measure. The nurse has already assessed the client's anxiety level. Use of defense mechanisms does not apply.

An intramuscular dose of antipsychotic medication needs to be administered to a client who is becoming increasingly more aggressive and refused to leave the day room. In what manner should the nurse enter the day room? a. Saying, "Would you like to come to your room and take some medication your health care provider prescribed for you?" b. Accompanied by three staff members and say, "Please come to your room so I can give you some medication that will help you regain control." c. Placing the client in a basket-hold and then saying, "I am going to take you to your room to give you an injection of medication to calm you." d. Being accompanied by a security guard and telling the client, "Come to your room willingly so I can give you this medication, or the guard and I will take you there."

b A client gains feelings of security if he or she sees others are present to help with control. The nurse gives a simple direction, honestly states what is going to happen, and reassures the client that the intervention will be helpful. This positive approach assumes the client can act responsibly and will maintain control. Physical control measures are used only as a last resort. The presence of the security guard is likely to intensive the client's agitation.

Which assessment finding presents the greatest risk for violent behavior directed at others? a. Severe agoraphobia b. History of spousal abuse c. Bizarre somatic delusions d. Verbalized hopelessness and powerlessness

b A history of prior aggression or violence is the best predictor of who may become violent. Clients with anxiety disorders are not particularly prone to violence unless panic occurs. Clients experiencing hopelessness and powerlessness may have coexisting anger, but violence is uncommon. Clients with paranoid delusions are at greater risk for violence than those with bizarre somatic delusions.

Police bring a client to the emergency department after an automobile accident. The client demonstrates poor coordination and slurred speech, but the vital signs are normal. The blood alcohol level is 300 mg/dL (0.30 g/dL). Considering the relationship between the assessment findings and blood alcohol level, which conclusion is most probable? a. The client rarely drinks alcohol. b. The client has a high tolerance to alcohol. c. The client has been treated with disulfiram. d. The client has ingested both alcohol and sedative drugs recently.

b A nontolerant drinker would have sleepiness and significant changes in vital signs with a blood alcohol level of 300 mg/dL (0.30 g/dL). The fact that the client is moving and talking shows a discrepancy between blood alcohol level and expected behavior and strongly indicates that the client's body is tolerant. If disulfiram and alcohol are ingested together, an entirely different clinical picture would result. The blood alcohol level gives no information about ingestion of other drugs.

A Client asks for information about alcoholics anonymous (AA). What is the nurse's best response? " a. AA is a form of group therapy led by a psychiatrist." b. AA is a self-help group for which the goal is sobriety." c. AA is a group that learns about drinking from a group leader." d. AA is a network that advocates strong punishment for drunk drivers."

b AA is a peer support group for recovering alcoholics. Neither professional nor peer leaders are appointed. AA does not advocate punishment but supports accountability for one's actions.

After the sudden death of his wife, a man says, "I can't live without her ... she was my whole life." What is the nurse's most therapeutic reply? a. "Each day will get a little better." b. "Her death is a terrible loss for you." c. "It's important to recognize that she is no longer suffering." d. "Your friends will help you cope with this change in your life."

b Adjustment disorders may be associated with grief. A statement that validates a bereaved person's loss is more helpful than false reassurances and clichés. It signifies understanding.

A Client diagnosed with alcohol use disorder asks, "How will Alcoholics Anonymous (AA) help me?" What is the nurse's best response? a. "The goal of AA is for members to learn controlled drinking with the support of a higher power." b. "An individual is supported by peers while striving for abstinence one day at a time." c. "You must make a commitment to permanently abstain from alcohol and other drugs." d. "You will be assigned a sponsor who will plan your treatment program."

b Admitting to being an alcoholic, making an attempt to remain alcohol-free for a day at a time, and receiving support from peers are basic aspects of AA. The other options are incorrect.

A client is experiencing moderate anxiety. The nurse encourages the client to talk about feelings and concerns. What is the rationale for this intervention? a. Offering hope allays and defuses the client's anxiety. b. Concerns stated aloud become less overwhelming and help problem solving begin. c. Anxiety is reduced by focusing on and validating what is occurring in the environment. d. Encouraging clients to explore alternatives increases the sense of control and lessens anxiety.

b All principles listed are valid, but the only rationale directly related to the intervention of assisting the client to talk about feelings and concerns is the one that states that concerns spoken aloud become less overwhelming and help problem solving begin.

A client performs ritualistic hand washing. Which action should the nurse implement to help the client develop more effective coping? a. Allow the client to set a hand-washing schedule. b. Encourage the client to participate in social activities. c. Encourage the client to discuss hand-washing routines. d. Focus on the client's symptoms rather than on the client.

b Because obsessive-compulsive clients become overly involved in the rituals, promotion of involvement with other people and activities is necessary to improve coping. Daily activities prevent constant focus on anxiety and symptoms. The other interventions focus on the compulsive symptom.

A woman is 5'7", 160 lbs. and wears a size 8 shoe. She says, "My feet are huge. I've asked three orthopedists to surgically reduce my feet." This person tries to buy shoes to make her feet look smaller and, in social settings, conceals both feet under a table or chair. Which health problem is likely? a. Social anxiety disorder b. Body dysmorphic disorder c. Separation anxiety disorder d. Obsessive-compulsive disorder due to a medical condition

b Body dysmorphic disorder refers to a preoccupation with an imagined defect in appearance in a normal-appearing person. The client's feet are proportional to the rest of the body. In obsessive-compulsive or related disorder due to a medical condition, the individual's symptoms of obsessions and compulsions are a direct physiological result of a medical condition. Social anxiety disorder, also called social phobia, is characterized by severe anxiety or fear provoked by exposure to a social or a performance situation that will be evaluated negatively by others. People with separation anxiety disorder exhibit developmentally inappropriate levels of concern over being away from a significant other.

A medical-surgical nurse works with a client diagnosed with a somatic symptom disorder. An understanding how what client characteristic will facilitate care planning? a. readily seek psychiatric counseling. b. be resistant to accepting psychiatric help. c. attend psychotherapy sessions without encouragement. d. be eager to discover the true reasons for physical symptoms.

b Clients diagnosed with somatic symptom disorders go from one health care provider to another trying to establish a physical cause for their symptoms. When a psychological basis is suggested and a referral for counseling offered, these clients reject both.

Which assessment data would help the health care team distinguish symptoms of conversion (functional neurological) disorder from symptoms of illness anxiety disorder (hypochondriasis)? a. Voluntary control of symptoms b. Client's style of presentation c. Results of diagnostic testing d. The role of secondary gains

b Clients with illness anxiety disorder (hypochondriasis) tend to be more anxious about their concerns and display more obsessive attention to detail, whereas the clients with conversion (functional neurological) disorder often exhibit less concern with the symptom they are presenting than would be expected. Neither disorder involves voluntary control of the symptoms. Results of diagnostic testing for both would be negative (i.e., no physiological basis would be found for the symptoms). Secondary gains can occur in both disorders but are not necessary to either. See relationship to audience response question.

A client reports fears of having cervical cancer and says to the nurse, "I've had Pap smears by six different doctors. The results were normal, but I'm sure that's because of errors in the laboratory." Which disorder would the nurse suspect? a. Conversion (functional neurological) disorder b. Illness anxiety disorder (hypochondriasis) c. Somatic symptom disorder d. Factitious disorder

b Clients with illness anxiety disorder have fears of serious medical problems, such as cancer or heart disease. These fears persist despite medical evaluations and interfere with daily functioning. There are no complaints of pain. There is no evidence of factitious or conversion disorder.

The unlicensed assistive personnel (UAP) says to the nurse, "That client with amnesia looks fine, but when I talk to her, she seems vague. What should I be doing for her?" What is the nurse's best reply? a. "Spend as much time with her as you can and ask questions about her life." b. "Use short, simple sentences and keep the environment calm and protective." c. "Provide more information about her past to reduce the mysteries that are causing anxiety." d. "Structure her time with activities to keep her busy, stimulated, and regaining concentration."

b Disruptions in ability to perform activities of daily living, confusion, and anxiety are often apparent in clients with amnesia. Offering simple directions to promote activities of daily living and reduce confusion helps increase feelings of safety and security. A calm, secure, predictable, protective environment is also helpful when a person is dealing with a great deal of uncertainty. Recollection of memories should proceed at its own pace, and the client should only gradually be given information about her past. Asking questions that require recall that the client does not possess will only add frustration. Quiet, undemanding activities should be provided as the client tolerates them and should be balanced with rest periods; the client's time should not be loaded with demanding or stimulating activities.

A nurse reviews vital signs for a client admitted with an injury sustained while intoxicated. The medical record shows these blood pressure and pulse readings at the times listed: 0200: 118/78 mm Hg and 72 beats/minute0400: 126/80 mm Hg and 76 beats/minute 0600: 128/82 mm Hg and 72 beats/minute 0800: 132/88 mm Hg and 80 beats/minute 1000: 148/94 mm Hg and 96 beats/minute What is the nurse's priority action? a. Force fluids. b. Begin the detox protocol. c. Obtain a clean-catch urine sample. d. Place the Client in a vest-type restraint.

b Elevated pulse and blood pressure may indicate impending alcohol withdrawal and the need for detox with medical intervention to prevent a hypertensive crisis and/or seizures. No indication is present that the Client may have a urinary tract infection or is presently in need of restraint. Hydration will not resolve the problem.

A new client acts out so aggressively that seclusion is required before the admission assessment is completed, or orders written. Immediately after safely secluding the client, which action is the nurse's priority? a. Complete the physical assessment. b. Notify the health care provider to obtain a seclusion order. c. Document the incident objectively in the client's medical record. d. Explain to the client that seclusion will be discontinued when self-control is regained.

b Emergency seclusion can be affected by a credentialed nurse but must be followed by securing a medical order within a period of time specified by the state and the agency. The incorrect options are not immediately necessary from a legal standpoint. See related audience response question.

A 15-year-old was placed in a residential program after truancy, running away, and an arrest for theft. At the program, the adolescent refused to join in planned activities and pushed a staff member, causing a fall. Which approach by nursing staff will be most therapeutic? a. Planned ignoring b. Establish firm limits c. Neutrally permit refusals d. Coaxing to gain compliance

b Firm limits are necessary to ensure physical safety and emotional security. Limit setting will also protect other patients from the teen's thoughtless or aggressive behavior. Permitting refusals to participate in the treatment plan, ignoring, coaxing, and bargaining are strategies that do not help the patient learn to abide by rules or structure.

A soldier returned home from active duty in a combat zone and was diagnosed with post- traumatic stress disorder (PTSD). The soldier says, "If there's a loud noise at night, I get under my bed because I think we're getting bombed." What type of experience has the soldier described? a. Illusion b. Flashback c. Nightmare d. Auditory hallucination

b Flashbacks are dissociative reactions in which an individual feels or acts as if the traumatic event were recurring. Illusions are misinterpretations of stimuli, and although the experience is similar, it is better termed a flashback because of the diagnosis of PTSD. Auditory hallucinations have no external stimuli. Nightmares commonly accompany PTSD, but this experience was stimulated by an actual environmental sound.

A client preparing for surgery has moderate anxiety and is unable to understand preoperative information. Which nursing intervention is most appropriate? a. Reassure the client that all nurses are skilled in providing postoperative care. b. Present the information again in a calm manner using simple language. c. Tell the client that staff is prepared to promote recovery. d. Encourage the client to express feelings to family.

b Giving information in a calm, simple manner will help the client grasp the important facts. Introducing extraneous topics as described in the distracters will further scatter the client's attention.

A client diagnosed with depersonalization disorder tells the nurse, "It's starting again. I feel as though I'm going to float away." Which intervention would be most appropriate at this point? a. Notify the health care provider of this change in the client's behavior. b. Engage the client in a physical activity such as exercise. c. Isolate the client until the sensation has diminished. d. Administer a prn dose of antianxiety medication.

b Helping the client apply a grounding technique, such as exercise, assists the client to interrupt the dissociative process. Medication can help reduce anxiety but does not directly interrupt the dissociative process. Isolation would allow the sensation to overpower the client. It is not necessary to notify the health care provider.

A client has a history of impulsively acting-out anger by physically striking others. What is the most appropriate intervention for avoiding similar incidents? a. Teach the client about herbal preparations that reduce anger. b. Help the client identify incidents that trigger impulsive anger. c. Explain that restraint and seclusion will be used if violence occurs. d. Offer one-on-one supervision to help the client maintain control.

b Identification of trigger incidents allows the client and nurse to plan interventions to reduce irritation and frustration, which lead to acting-out anger, and eventually to put into practice more adaptive coping strategies. None of the other options address the cause of the anger.

The nurse assesses a client who reports loneliness and episodes of anxiety. Which statement by the client is mostly likely if this client also has agoraphobia? a. "I'm sure I will get over not wanting to leave home soon. It takes time." b. "Being afraid to go out seems ridiculous, but I can't go out the door." c. "My family says they like it now that I stay home most of the time." d. "When I have a good incentive to go out, I can do it."

b Individuals who are agoraphobic generally acknowledge that the behavior is not constructive and that they do not really like it. The symptom is ego dystonic. However, clients will state they are unable to change the behavior. Agoraphobics are not optimistic about change. Most families are dissatisfied when family members refuse to leave the house.

A client experiences a sudden episode of severe anxiety. Of these medications in the client's medical record, which is most appropriate to give as a prn anxiolytic? a. buspirone b. lorazepam c. amitriptyline d. desipramine

b Lorazepam is a benzodiazepine used to treat anxiety. It may be given as a prn medication. Buspirone is long acting and is not useful as a prn drug. Amitriptyline and desipramine are tricyclic antidepressants and considered second- or third-line agents.

A client with an abdominal mass is scheduled for a biopsy. The client has difficulty understanding the nurse's comments and asks, "What do you mean? What are they going to do?" Assessment findings include tremulous voice, respirations 28, and pulse 110. What is the client's level of anxiety? a. Mild b. Moderate c. Severe d. Panic

b Moderate anxiety causes the individual to grasp less information and reduces problem-solving ability to a less-than-optimal level. Mild anxiety heightens attention and enhances problem solving. Severe anxiety causes great reduction in the perceptual field. Panic-level anxiety results in disorganized behavior.

A child known as the neighborhood bully says, ―Nobody can tell me what to do.‖ After receiving a poor grade on a science project, this child secretly loaded a virus on the teacher's computer. These behaviors support a diagnosis of a. CD. b. ODD. c. intermittent explosive disorder. d. ADHD.

b ODD is a repeated and persistent pattern of having an angry and irritable mood in conjunction with demonstrating defiant and vindictive behavior. Loading a virus is a vindictive behavior in retribution for a poor grade. Persons with CD are aggressive against people and animals; destroy property; are deceitful; violate rules; and have impaired social, academic, or occupational functioning. There is no evidence of explosiveness or distractibility.

A hospitalized Client diagnosed with alcohol use disorder believes spiders are spinning entrapping webs in the room. The client is fearful, agitated, and diaphoretic. Which nursing intervention is indicated? a. Check the client every 15 minutes b. One-on-one supervision c. Keep the room dimly lit d. Force fluids

b One-on-one supervision is necessary to promote physical safety until sedation reduces the Client's feelings of terror. Checks every 15 minutes would not be sufficient to provide for safety. A dimly lit room promotes perceptual disturbances. Excessive fluid intake can cause overhydration, because fluid retention normally occurs when blood alcohol levels fall.

A nurse cares for a client experiencing an opioid overdose. Which focused assessment has the highest priority? a. Cardiovascular b. Respiratory c. Neurological d. Hepatic

b Opioid overdose causes respiratory depression. Respiratory depression is the primary cause of death among opioid abusers. The assessment of the other body systems is relevant but not the priority.

A nurse works with a client diagnosed with posttraumatic stress disorder (PTSD) who has frequent flashbacks as well as persistent symptoms of arousal. Which intervention should be included in the plan of care? a. Trigger flashbacks intentionally in order to help the client learn to cope with them. b. Explain that the physical symptoms are related to the psychological state. c. Encourage repression of memories associated with the traumatic event. d. Support "numbing" as a temporary way to manage intolerable feelings.

b Persons with PTSD often experience somatic symptoms or sympathetic nervous system arousal that can be confusing and distressing. Explaining that these are the body's responses to psychological trauma helps the client understand how such symptoms are part of the illness and something that will respond to treatment. This decreases powerlessness over the symptoms and helps instill a sense of hope. It also helps the client to understand how relaxation, breathing exercises, and imagery can be helpful in symptom reduction. The goal of treatment for PTSD is to come to terms with the event so treatment efforts would not include repression of memories or numbing. Triggering flashbacks would increase client distress.

A client experiences blindness related to conversion (functional neurological) disorder but is unconcerned about this problem. Which understanding should guide the nurse's planning for this client? a. The client is suppressing accurate feelings regarding the problem. b. The client's anxiety is relieved through the physical symptom. c. The client's optic nerve transmission has been impaired. d. The client will not disclose genuine fears.

b Psychoanalytical theory suggests conversion reduces anxiety through production of a physical symptom symbolically linked to an underlying conflict. Conversion, not suppression, is the operative defense mechanism in this disorder. While some MRI studies suggest that clients with conversion disorder have an abnormal pattern of cerebral activation, there is no actual alternation of nerve transmission. The other distracters oversimplify the dynamics, suggesting that only dependency needs are of concern, or suggest conscious motivation (conversion operates unconsciously).

A client says, "I know I have a brain tumor despite the results of the magnetic resonance imaging (MRI). The radiologist is wrong. People who have brain tumors vomit, and yesterday I vomited all day." Which response by the nurse fosters cognitive reframing? a. "You do not have a brain tumor. The more you talk about it, the more it reinforces your belief." b. "Let's see if there are any other possible explanations for your vomiting." c. "You seem so worried. Let's talk about how you're feeling." d. "We need to talk about something else."

b Questioning the evidence is a cognitive reframing technique. Identifying causes other than the feared disease can be helpful in changing distorted perceptions. Distraction by changing the subject will not be effective.

An 11-year-old diagnosed with ODD becomes angry over the rules at a residential treatment program and begins shouting at the nurse. What is the nurse's initial action to defuse the situation? a. Say to the child, ―Tell me how you're feeling right now.‖ b. Take the child swimming at the facility's pool. c. Establish a behavioral contract with the child. d. Administer an anxiolytic medication.

b Redirecting the expression of feelings into nondestructive, age-appropriate behaviors such as a physical activity helps the child learn how to modulate the expression of feelings and exert self-control. This is the least restrictive alternative and should be tried before resorting to measures that are more restrictive. A shouting child will not likely engage in a discussion about feelings. A behavioral contract could be considered later, but first the situation must be defused.

A hospitalized client diagnosed with alcohol use disorder believes the window blinds are snakes trying to get in the room. The client is anxious, agitated, and diaphoretic. The nurse can anticipate the health care provider will prescribe what medication intervention? a. narcotic analgesic, such as hydromorphone. b. sedative, such as lorazepam or chlordiazepoxide. c. antipsychotic, such as olanzapine or thioridazine. d. monoamine oxidase inhibitor antidepressant, such as phenelzine.

b Sedation allows for safe withdrawal from alcohol. Benzodiazepines are the drugs of choice in most regions because of their high therapeutic safety index and anticonvulsant properties. The client's highest needs related to a need for calming.

A client is experiencing blindness related to conversion (functional neurological) disorder. To help the client eat, the nurse should implement which intervention? a. establish a "buddy" system with other clients who can feed the client at each meal. b. expect the client to feed self after explaining arrangement of the food on the tray. c. direct the client to locate items on the tray independently and feed self. d. address needs of other clients in the dining room, then feed this client.

b The client is expected to maintain some level of independence by feeding self, while the nurse is supportive in a matter-of-fact way. The distracters support dependency or offer little support.

A client experiencing blindness related to conversion (functional neurological) disorder states, "All the doctors and nurses in the hospital stop by often to check on me. Too bad people outside the hospital don't find me as interesting." Which nursing diagnosis is most relevant? a. Social isolation b. Chronic low self-esteem c. Interrupted family processes d. Ineffective health maintenance

b The client mentions that the symptoms make people more interested. This indicates that the client feels uninteresting and unpopular without the symptoms, thus supporting the nursing diagnosis of chronic low self-esteem. Defining characteristics for the other nursing diagnoses are not present in the scenario.

An adult in the emergency department states, "Everything I see appears to be waving. I am outside my body looking at myself. I think I'm losing my mind." When vital signs are slightly elevated what should the nurse suspect? a. a schizophrenic episode. b. hallucinogen ingestion. c. opium intoxication. d. cocaine overdose.

b The client who is high on a hallucinogen often experiences synesthesia (visions in sound), depersonalization, and concerns about going "crazy." Synesthesia is not common in schizophrenia. CNS stimulant overdose more commonly involves elevated vital signs and assaultive, grandiose behaviors. Phencyclidine (PCP) use commonly causes bizarre or violent behavior, nystagmus, elevated vital signs, and repetitive jerking movements.

A client with a somatic symptom disorder has the nursing diagnosis Interrupted family processes related to client's disabling symptoms as evidenced by spouse and children assuming roles and tasks that previously belonged to client. What is an appropriate outcome for this client? a. Will assume roles and functions of other family members. b. Will demonstrate performance of former roles and tasks. c. Will focus energy on problems occurring in the family. d. Will rely on family members to meet personal needs.

b The client with a somatic symptom disorder has typically adopted a sick role in the family, characterized by dependence. Increasing independence and resumption of former roles are necessary to change this pattern. The distracters are inappropriate outcomes.

Which comment by the parents of young children best demonstrates support of development of resilience and effective stress management? a. "Our children will be stronger if they make their own decisions." b. "We spend daily family time talking about experiences and feelings." c. "We use three different babysitters. All of them have college degrees." d. "Our parenting strategies are different from those our own parents used."

b The correct response demonstrates consistent nurturing, which is a vital component of building resilience in children. The incorrect responses are not necessarily unhealthy parenting behaviors, but they do not clearly demonstrate parental nurturing.

An adult client assaulted another client and was then restrained. One hour later, which statement by the restrained client requires the nurse's immediate attention? a. "I hate all of you!" b. "My fingers are tingly." c. "You wait until I tell my lawyer." d. "The other client started the fight."

b The correct response indicates impaired circulation and necessitates the nurse's immediate attention. The incorrect responses indicate the client has continued aggressiveness and agitation.

Two weeks ago, a soldier returned to the United States from active duty in a combat zone. The soldier was diagnosed with post-traumatic stress disorder (PTSD). Which comment by the soldier requires the nurse's immediate attention? a. "It's good to be home. I missed my home, family, and friends." b. "I saw my best friend get killed by a roadside bomb. I don't understand why it wasn't me." c. "Sometimes I think I hear bombs exploding, but it's just the noise of traffic in my hometown." d. "I want to continue my education, but I'm not sure how I will fit in with other college students."

b The correct response indicates the soldier is thinking about death and feeling survivor's guilt. These emotions may accompany suicidal ideation, which warrants the nurse's follow-up assessment. Suicide is a high risk among military personnel diagnosed with PTSD. One distracter indicates flashbacks, common with persons with PTSD, but not solely indicative that further problems exist. The other distracters are normal emotions associated with returning home and change.

In the emergency department, a client's vital signs are BP 66/40 mm Hg; pulse 140 beats/minute; respirations 8 breaths/minute and shallow. The nursing diagnosis is Ineffective breathing pattern related to depression of respiratory center secondary to opioid intoxication. What is the priority outcome? a. The client will demonstrate effective coping skills and identify community resources for treatment of substance abuse within 1 week of hospitalization. b. Within 4 hours, vital signs will stabilize, with BP above 90/60 mm Hg, pulse less than 100 beats/minute, and respirations at or above 12 breaths/minute. c. The client will correctly describe a plan for home care and achieving a drug-free state before release from the emergency department. d. Within 6 hours, the client's breath sounds will be clear bilaterally and throughout lung fields.

b The correct short-term outcome is the only one that relates to the client's physical condition. It is expected that vital signs will return to normal when the CNS depression is alleviated. The client's respirations are slow and shallow, but there is no evidence of congestion.

The gas pedal on a person's car became stuck on a busy interstate highway, causing the car to accelerate rapidly. For 20 minutes, the car was very difficult to control. In the months after this experience, afterward, which assessment finding would the nurse expect? a. Weight gain b. Flashbacks c. Headache d. Diuresis

b The scenario depicts a frightening, traumatic, and stressful situation. Severe dissociation or "mind flight" may occur for those who have suffered significant trauma. The episodic failure of dissociation causes intrusive symptoms such as flashbacks. The problems identified in the distracters may or may not occur.

Which symptoms of withdrawal from opioids should the nurse assess for? a. dilated pupils, tachycardia, elevated blood pressure, and elation. b. nausea, vomiting, diaphoresis, anxiety, and hyperreflexia. c. mood lability, incoordination, fever, and drowsiness. d. excessive eating, constipation, and headache.

b The symptoms of withdrawal from opioids are similar to those of alcohol withdrawal. Hyperthermia is likely to produce periods of diaphoresis.

A client with a history of anger and impulsivity was hospitalized after an accident resulting in multiple injuries. The client loudly scolds nursing staff, "I'm in pain all the time but you don't give me medicine until YOU think it's time." Which nursing intervention would best address this problem? a. Teach the client to use coping strategies such as deep breathing and progressive relaxation to reduce the pain. b. Talk with the health care provider about changing the pain medication from prn to client-controlled analgesia. c. Tell the client that verbal assaults on nurses will not shorten the wait for analgesic medication. d. Talk with the client about the risks of dependency associated with overuse of analgesic medication.

b Use of client-controlled analgesia will help the client manage the pain. This intervention will help reduce the client's anxiety and anger. Dependency is not an important concern related to acute pain.

Which prescription medication would the nurse expect to be prescribed for a client diagnosed with a somatic symptom disorder? a. Narcotic analgesics for use as needed for acute pain b. Antidepressant medications to treat co-morbid depression c. Long-term use of benzodiazepines to support coping with anxiety d. Conventional antipsychotic medications to correct cognitive distortions

b Various types of antidepressants may be helpful in somatic disorders not only directly by reducing depressive symptoms and hence somatic responses, but also indirectly by affecting nerve circuits that affect not only mood but also fatigue, pain perception, GI distress, and other somatic symptoms. Clients may benefit from short-term use of antianxiety medication (benzodiazepines) but require careful monitoring because of risks of dependence. Conventional antipsychotic medications would not be used, although selected atypical antipsychotics may be useful. Narcotic analgesics are not indicated.

A client undergoing alcohol rehabilitation decides to begin disulfiram therapy. Client teaching should include the need to (Select all that apply.) a. avoid aged cheeses. b. avoid alcohol-based skin products. c. read labels of all liquid medications. d. wear sunscreen and avoid bright sunlight. e. maintain an adequate dietary intake of sodium. f. avoid breathing fumes of paints, stains, and stripping compounds.

b, c, f The Client must avoid hidden sources of alcohol. Many liquid medications, such as cough syrups, contain small amounts of alcohol that could trigger an alcohol-disulfiram reaction. Using alcohol-based skin products such as aftershave or cologne, smelling alcohol-laden fumes, and eating foods prepared with wine, brandy, or beer may also trigger reactions. The other options do not relate to hidden sources of alcohol.

A nurse works with an adolescent who was placed in a residential program after multiple episodes of violence at school. Establishing rapport with this adolescent is a priority because (Select all that apply) a. it is a vital component of implementing a behavior modification program. b. a therapeutic alliance is the first step in a nurse's therapeutic use of self. c. the adolescent has demonstrated resistance to other authority figures. d. acceptance and trust convey feelings of security for the adolescent. e. adolescents usually relate better to authority figures than peers.

b, d Trust is frequently an issue because the adolescent may never have had a trusting relationship with an adult. Trust promotes feelings of security and is the basis of the nurse's therapeutic use of self. Adolescents value peer relationships over those related to authority. Rewards for appropriate behavior are the main component of behavior modification programs.

A child has a history of multiple hospitalizations for recurrent systemic infections. The child is not improving in the hospital, despite aggressive treatment. Factitious disorder imposed on another is suspected. Which nursing interventions are appropriate? (Select all that apply.) a. Increase private visiting time for the parents to improve bonding. b. Keep careful, detailed records of visitation and untoward events. c. Place mittens on the child to reduce access to ports and incisions. d. Encourage family members to visit in groups of two or three. e. Interact with the client frequently during visiting hours.

b, d, e Factitious disorder imposed on another is a condition wherein a person intentionally causes or perpetuates the illness of a loved one (e.g., by periodically contaminating IV solutions with fecal material). When this disorder is suspected, the child's life could be at risk. Depending on the evidence supporting this suspicion, interventions could range from minimizing unsupervised visitation to blocking visitation altogether. Frequently checking on the child during visitation and minimizing unobserved access to the child (by encouraging small group visits) reduces the opportunity to take harmful action and increases the collection of data that can help determine whether this disorder is at the root of the child's illness. Detailed tracking of visitation and untoward events helps identify any patterns there might be between select visitors and the course of the child's illness. Increasing private visitation provides more opportunity for harm. Educating visitors about aseptic techniques would not be of help if the infections are intentional and preventing inadvertent contamination by the child himself would not affect factitious disorder by proxy.

Which assessment questions would be most appropriate for the nurse to ask a client with possible obsessive-compulsive disorder? (Select all that apply.) a. "Are there certain social situations that cause you to feel especially uncomfortable?" b. "Are there others in your family who must do things in a certain way to feel comfortable?" c. "Have you been a victim of a crime or seen someone badly injured or killed?" d. "Is it difficult to keep certain thoughts out of your awareness?" e. "Do you do certain things over and over again?"

b, d, e The correct questions refer to obsessive thinking and compulsive behaviors. There is likely a genetic correlation to the disorder. The incorrect responses are more pertinent to a client with suspected posttraumatic stress disorder or with suspected social phobia.

What are the primary distinguishing factors between the behavior of persons diagnosed with ODD and those with CD? The person diagnosed with (Select all that apply) a. ODD relives traumatic events by acting them out. b. ODD tests limits and disobeys authority figures. c. ODD has difficulty separating from loved ones. d. CD uses stereotypical or repetitive language. e. CD often violates the rights of others.

b, e Persons diagnosed with ODD are negativistic, disobedient, and defiant toward authority figures without seriously violating the basic rights of others, whereas persons with CD frequently behave in ways that do violate the rights of others and age-appropriate societal norms. Reliving traumatic events occurs with PTSD. Stereotypical language behaviors are seen in persons with autism spectrum disorders.

The treatment team discusses the plan of care for a client diagnosed with schizophrenia and daily cannabis abuse who is having increased hallucinations and delusions. To plan effective treatment, the team should consider what intervention? a. provide long-term care for the client in a residential facility. b. withdraw the client from cannabis, then treat the schizophrenia. c. consider each diagnosis primary and provide simultaneous treatment. d. first treat the schizophrenia, then establish goals for substance abuse treatment.

c Both diagnoses should be considered primary and receive simultaneous treatment. Comorbid disorders require longer treatment and progress is slower, but treatment may occur in the community.

Which clinical scenario predicts the highest risk for directing violent behavior toward others? a. Major depressive disorder with delusions of worthlessness b. Obsessive-compulsive disorder; performs many rituals c. Paranoid delusions of being followed by alien monsters d. Completed alcohol withdrawal; beginning a rehabilitation program

c Clients who are delusional, hyperactive, impulsive, or predisposed to irritability are at higher risk for violence. The client in the correct response has the greatest disruption of ability to perceive reality accurately. People who feel persecuted may strike out against those believed to be persecutors. The other clients have better reality-testing ability.

An adolescent diagnosed with a CD stole and wrecked a neighbor's motorcycle. Afterward, the adolescent was confronted about the behavior but expressed no remorse. Which variation in the central nervous system best explains the adolescent's reaction? a. Serotonin dysregulation and increased testosterone activity impair one's capacity for remorse. b. Increased neuron destruction in the hippocampus results in decreased abilities to conform to social rules. c. Reduced gray matter in the cortex and dysfunction of the amygdala results in decreased feelings of empathy. d. Disturbances in the occipital lobe reduce sensations that help an individual clearly visualize the consequences of behavior.

c Adolescents with CD have been found to have significantly reduced gray matter bilaterally in the anterior insulate cortex and the amygdala. This reduction may be related to aggressive behavior and deficits of empathy. The less gray matter in these regions of the brain, the less likely adolescents are to feel remorse for their actions or victims. People with intermittent explosive disorder may have differences in serotonin regulation in the brain and higher levels of testosterone. Neuron destruction in the hippocampus is associated with memory deficits. The occipital lobe is involved with visual stimuli but not the processing of emotions.

Family members describe the client as "a difficult person who finds fault with others." The client verbally abuses nurses for their poor care. What is the most likely explanation for this behavior? a. poor childrearing that did not teach respect for others. b. automatic thinking leading to cognitive distortions. c. a personality style that externalizes problems. d. delusions that others wish to deliver harm.

c Clients whose personality style causes them to externalize blame see the source of their discomfort and anxiety as being outside themselves. They displace anger and are often unable to self-soothe. The incorrect options are less likely to have a bearing on this behavior.

A soldier returns to the United States from active duty in a combat zone. The soldier is diagnosed with post-traumatic stress disorder (PTSD). The nurse's highest priority is to screen this soldier for a. bipolar disorder. b. schizophrenia. c. depression. d. dementia.

c Comorbidities for adults with PTSD include depression, anxiety disorders, sleep disorders, and dissociative disorders. Incidence of the disorders identified in the distracters is similar to the general population.

A client diagnosed with a somatic symptom disorder has been in treatment for 4 weeks. The client says, "Although I'm still having pain, I notice it less and am able to perform more activities." The nurse should evaluate the treatment plan as at what degree of success? a. marginally b. minimally c. partially d. totally

c Decreased preoccupation with symptoms and increased ability to perform activities of daily living suggest partial success of the treatment plan. Total success is rare because of client resistance.

A client states, "I feel detached and weird all the time. It is as though I am looking at life through a cloudy window. Everything seems unreal. It really messes up things at work and school." This scenario is most suggestive of which health problem? a. Acute stress disorder b. Dissociative amnesia c. Depersonalization disorder d. Disinhibited social engagement disorder

c Depersonalization disorder involves a persistent or recurrent experience of feeling detached from and outside oneself. Although reality testing is intact, the experience causes significant impairment in social or occupational functioning and distress to the individual. Dissociative amnesia involves memory loss. Children with disinhibited social engagement disorder demonstrate no normal fear of strangers and are unusually willing to go off with strangers. Individuals with ASD (Acute Stress Disorder) experience three or more dissociative symptoms associated with a traumatic event, such as a subjective sense of numbing, detachment, or absence of emotional responsiveness; a reduction in awareness of surroundings; derealization; depersonalization or dissociative amnesia. In the scenario, the client experiences only one symptom.

Shortly after the parents announced that they were divorcing, a 15-year-old became truant from school and assaulted a friend. The adolescent told the school nurse, ―I'd rather stay in my room and listen to music. It's easier than thinking about what is happening in my family.‖ Which nursing diagnosis is most applicable? a. Chronic low self-esteem related to role within the family b. Decisional conflict related to compliance with school requirements c. Defensive coping related to adjustment to changes in family relationships d. Disturbed personal identity related to self-perceptions of changing family dynamics

c Depression is often associated with impulse control disorder. The correct nursing diagnosis refers to the patient's dysfunctional management of feelings associated with upcoming changes to the family. The teen displays self-imposed isolation. The distracters are not supported by data in the scenario.

When alprazolam is prescribed for a client who experiences acute anxiety, health teaching should include which instruction? a. report drowsiness. b. eat a tyramine-free diet. c. avoid alcoholic beverages. d. adjust dose and frequency based on anxiety level.

c Drinking alcohol or taking other anxiolytics along with the prescribed benzodiazepine should be avoided because depressant effects of both drugs will be potentiated. Tyramine-free diets are necessary only with monoamine oxidase inhibitors (MAOIs). Drowsiness is an expected effect and needs to be reported only if it is excessive. Clients should be taught not to deviate from the prescribed dose and schedule for administration.

A client experiencing moderate anxiety says, "I feel undone." What would be the appropriate response by the nurse? a. "What would you like me to do to help you?" b. "Why do you suppose you are feeling anxious?" c. "I'm not sure I understand. Give me an example." d. "You must get your feelings under control before we can continue."

c Increased anxiety results in scattered thoughts and an inability to articulate clearly. Clarifying helps the client identify thoughts and feelings. Asking the client why he or she feels anxious is nontherapeutic; the client likely does not have an answer. The client may be unable to determine what he or she would like the nurse to do in order to help. Telling the client to get his or her feelings under control is a directive the client is probably unable to accomplish.

A client is pacing the hall near the nurses' station, swearing loudly. An appropriate initial intervention for the nurse would be to address the client by name and to make what statement? a. "What is going on?" b. "Please be quiet and sit down in this chair immediately." c. "I'd like to talk with you about how you're feeling right now." d. "You must go to your room and try to get control of yourself."

c Intervention should begin with analysis of the client and the situation. When anger is escalating, a client's ability to process decreases. It is important to speak to the client slowly and in short sentences, using a low and calm voice. Use open-ended statements designed to hear the client's feelings and concerns. This leads to the next step of planning an intervention.

A client diagnosed with obsessive-compulsive disorder has this nursing diagnosis: Anxiety related to ____________________ as evidenced by inability to control compulsive cleaning. Which phrase correctly completes the etiological portion of the diagnosis? a. feelings of responsibility for the health of family members b. approval-seeking behavior from friends and family c. persistent thoughts about bacteria, germs, and dirt d. needs to avoid interactions with others

c Many compulsive rituals accompany obsessive thoughts. The client uses these rituals for anxiety relief. Unfortunately, the anxiety relief is short lived, and the client must frequently repeat the ritual. The other options are unrelated to the dynamics of compulsive behavior.

A client has smoked two packs of cigarettes daily for many years. When the client tries to reduce smoking, anxiety, craving, poor concentration, and headache occur. This scenario should be described using which term? a. cross-tolerance. b. substance abuse. c. substance addiction. d. substance intoxication.

c Nicotine meets the criteria for a "substance," the criterion for addiction is present, and withdrawal symptoms are noted with abstinence or reduction of dose. The scenario does not meet criteria for substance abuse, intoxication, or cross-tolerance.

Which prescribed medication should a nurse administer to provide immediate intervention for a psychotic client whose aggressive behavior continues to escalate despite verbal intervention? a. Lithium b. Trazodone c. Olanzapine d. Valproic acid

c Olanzapine is a short-acting antipsychotic useful in calming angry, aggressive clients regardless of diagnosis. The other drugs listed require long-term use to reduce anger. Lithium is for bipolar clients. Trazodone is commonly prescribed for clients experiencing depression, insomnia, or chronic pain. Valproic acid is for bipolar or borderline clients.

An emergency department nurse realizes that the spouse of a client is becoming increasingly irritable while waiting. Which intervention should the nurse use to prevent further escalation of the spouse's anger? a. Offer the waiting spouse a cup of coffee. b. Explain that the client's condition is not life threatening. c. Periodically provide an update and progress report on the client. d. Suggest that the spouse return home until the client's treatment is complete.

c Periodic updates reduce anxiety and defuse anger. This strategy acknowledges the spouse's presence and concern. A cup of coffee is a nice gesture, but it does not address the spouse's feelings. The other incorrect options would be likely to increase anger because they imply that the anxiety is inappropriate.

A soldier who served in a combat zone returned to the United States. The soldier's spouse complains to the nurse, "We had planned to start a family, but now he won't talk about it. He won't even look at children." The spouse is describing which symptom associated with post- traumatic stress disorder (PTSD)? a. Re-experiencing b. Hyperarousal c. Avoidance d. Psychosis

c Physiological reactions to reminders of the event that include persistent avoidance of stimuli associated with the trauma results in the individual's avoiding talking about the event or avoiding activities, people, or places that arouse memories of the trauma. Avoidance is exemplified by a sense of foreshortened future and estrangement. There is no evidence this soldier is having hyperarousal or reexperiencing war-related traumas. Psychosis is not evident.

Two staff nurses applied for a charge nurse position. After the promotion was announced, the nurse who was not promoted said, "The nurse manager had a headache the day I was interviewed." Which defense mechanism is evident? a. Introjection b. Conversion c. Projection d. Splitting

c Projection is the hallmark of blaming, scapegoating, prejudicial thinking, and stigmatizing others. Conversion involves the unconscious transformation of anxiety into a physical symptom. Introjection involves intense, unconscious identification with another person. Splitting is the inability to integrate the positive and negative qualities of oneself or others into a cohesive image.

A client tells a nurse, "My best friend is a perfect person. She is kind, considerate, good- looking, and successful with every task. I could have been like her if I had the opportunities, luck, and money she's had." This client is demonstrating a. denial. b. projection. c. rationalization. d. compensation.

c Rationalization consists of justifying illogical or unreasonable ideas, actions, or feelings by developing acceptable explanations that satisfy the teller as well as the listener. Denial is an unconscious process that would call for the nurse to ignore the existence of the situation. Projection operates unconsciously and would result in blaming behavior. Compensation would result in the nurse unconsciously attempting to make up for a perceived weakness by emphasizing a strong point.

A cruel and abusive person often uses rationalization to explain the behavior. Which comment demonstrates use of this defense mechanism? a. "I don't know why I do mean things." b. "I have always had poor impulse control." c. "That person should not have provoked me." d. "I'm really a coward who is afraid of being hurt."

c Rationalization consists of justifying one's unacceptable behavior by developing explanations that satisfy the teller and attempt to satisfy the listener. The abuser is suggesting that the abuse is not his or her fault; it would not have occurred except for the provocation by the other person. The distracters indicate some measure of acceptance of responsibility for the behavior.

A client experiencing panic suddenly began running and shouting, "I'm going to explode!" What is the nurse's best action? a. Ask, "I'm not sure what you mean. Give me an example." b. Capture the client in a basket-hold to increase feelings of control. c. Tell the client, "Stop running and take a deep breath. I will help you." d. Assemble several staff members and say, "We will take you to seclusion to help you regain control."

c Safety needs of the client and other clients are a priority. Comments to the client should be simple, neutral, and give direction to help the client regain control. Running after the client will increase the client's anxiety. More than one staff member may be needed to provide physical limits but using seclusion or physically restraining the client prematurely is unjustified. Asking the client to give an example would be futile; a client in panic processes information poorly.

A client diagnosed with a somatic symptom disorder says, "My pain is from an undiagnosed injury. I can't take care of myself. I need pain medicine six or seven times a day. I feel like a baby because my family has to help me so much." What is the most important nursing assessment? a. mood. b. cognitive style. c. secondary gains. d. identity and memory.

c Secondary gains should be assessed. The client's dependency needs may be met through care from the family. When secondary gains are prominent, the client is more resistant to giving up the symptom. The scenario does not allude to a problem of mood. Cognitive style and identity and memory assessment are of lesser concern because the client's diagnosis has been established.

An adolescent acts out in disruptive ways. When this adolescent threatens to throw a heavy pool ball at another adolescent, which comment by the nurse would set appropriate limits? a. Attention everyone: we are all going to the craft room b. You will be taken to seclusion if you throw that ball. c. Do not throw the ball. Put it back on the pool table d. Please do not lose control of your emotions.

c Setting limits uses clear, sharp statements about prohibited behavior and guidance for performing a behavior that is expected. The incorrect options represent a threat, use of restructuring (which would be inappropriate in this instance), and a direct appeal to the child's developing self-control that may be ineffective.

To assist clients diagnosed with somatic symptom disorders, which nursing interventions have the highest priority? a. explain the pathophysiology of symptoms. b. help these clients suppress feelings of anger. c. shift focus from somatic symptoms to feelings. d. investigate each physical symptom as it is reported.

c Shifting the focus from somatic symptoms to feelings or to neutral topics conveys interest in the client as a person rather than as a condition. The need to gain attention with the use of symptoms is reduced over the long term. A desired outcome would be that the client would express feelings, including anger if it is present. Once physical symptoms are investigated, they do not need to be reinvestigated each time the client reports them.

Which treatment modality should a nurse recommend to help a client diagnosed with a somatic symptom disorder to cope more effectively? a. Flooding b. Response prevention c. Relaxation techniques d. Systematic desensitization

c Somatic symptom disorders are commonly associated with complicated reactions to stress. These reactions are accompanied by muscle tension and pain. Relaxation can diminish the client's perceptions of pain and reduce muscle tension. The distracters are modalities useful in treating selected anxiety disorders.

A client with multi-infarct dementia lashes out and kicks at people who walk past in the hall of a skilled nursing facility. Intervention by the nurse should begin with what intervention? a. gently touching the client's arm. b. asking the client, "What do you need?" c. saying to the client, "This is a safe place." d. directing the client to cease the behavior.

c Striking out usually signals fear or that the client perceives the environment to be out of control. Getting the client's attention is fundamental to intervention. The nurse should make eye contact and assure the client of safety. Once the nurse has the client's attention, gently touching the client, asking what he or she needs, or directing the client to discontinue the behavior may be appropriate.

A woman just received notification that her husband died. She approaches the nurse who cared for him during his last hours and says angrily, "If you had given him your undivided attention, he would still be alive." How should the nurse analyze this behavior? a. The comment suggests potential allegations of malpractice. b. In some cultures, grief is expressed solely through anger. c. Anger is an expected emotion in an adjustment disorder. d. The client had ambivalent feelings about her husband.

c Symptoms of adjustment disorder run the gamut of all forms of distress including guilt, depression, and anger. Anger may protect the bereaved from facing the devastating reality of loss.

A client admitted for injuries sustained while intoxicated has been hospitalized for 48 hours. The client is now shaky, irritable, anxious, diaphoretic, and reports nightmares. The pulse rate is 130 beats/minute. The client shouts, "Bugs are crawling on my bed. I've got to get out of here." What is the most accurate assessment of this situation? a. The client is attempting to obtain attention by manipulating staff. b. The client may have sustained a head injury before admission. c. The client has symptoms of alcohol withdrawal delirium. d. The client is having an acute psychosis.

c Symptoms of agitation, elevated pulse, and perceptual distortions indicate alcohol withdrawal delirium. The findings are inconsistent with manipulative attempts, head injury, or functional psychosis.

A person runs from a crowded nightclub after a pyrotechnics show causes the building to catch fire. Which division of the autonomic nervous system will be stimulated in response to this experience? a. Limbic system b. Peripheral nervous system c. Sympathetic nervous system d. Parasympathetic nervous system

c The autonomic nervous system is comprised of the sympathetic (fight or flight response) and parasympathetic nervous system (relaxation response). In times of stress, the sympathetic nervous system is stimulated. A person would experience stress associated with the experience of being in danger. The peripheral nervous system responds to messages from the sympathetic nervous system. The limbic system processes emotional responses but is not specifically part of the autonomic nervous system.

An 11-year-old diagnosed with ODD becomes angry over the rules at a residential treatment program and begins cursing at the nurse. Select the best method for the nurse to defuse the situation. a. Ignore the child's behavior. b. Send the child to time-out for 2 hours. c. Take the child to the gym and engage in an activity. d. Role-play a more appropriate behavior with the child.

c The child's behavior warrants an active response. Redirecting the expression of feelings into nondestructive age-appropriate behaviors, such as a physical activity, helps defuse the situation here and now. This response helps the child learn how to modulate the expression of feelings and exert self-control. This is the least restrictive alternative and should be tried before resorting to a more restrictive measure. Role playing is appropriate after the child's anger is defused.

After an assault by a client, a nurse has difficulty sleeping, startles easily, and is preoccupied with the incident. The nurse said, "That client should not be allowed to get away with that behavior." Which response poses the greatest barrier to the nurse's ability to provide therapeutic care? a. Startle reactions b. Difficulty sleeping c. A wish for revenge d. Preoccupation with the incident

c The desire for revenge signals an urgent need for professional supervision to work through anger and counter the aggressive feelings. Feelings of revenge create a risk for harm to the client. The distracters are normal in a person who was assaulted. They usually are relieved with crisis intervention, help the individual regain a sense of control, and make sense of the event.

A nurse prepares for an initial interaction with a client with a long history of methamphetamine abuse. Which is the nurse's best first action? a. Perform a thorough assessment of the client. b. Verify that security services are immediately available. c. Self-assess personal attitude, values, and beliefs about this health problem. d. Obtain a face shield because oral hygiene is poor in methamphetamine abusers.

c The nurse should show compassion, care, and helpfulness for all clients, including those with addictive diseases. It is important to have a clear understanding of one's own perspective. Negative feelings may occur for the nurse; supervision is an important resource. The activities identified in the distracters occur after self-assessment.

An adolescent was recently diagnosed with ODD. The parents say to the nurse, ―Isn't there some medication that will help with this problem? Select the nurse's best response. a. There are no medications to treat this problem. This diagnosis is behavioral in nature. b. It's a common misconception that there is a medication available to treat every health problem. c. Medication is usually not prescribed for this problem. Let's discuss some behavioral strategies you can use. d. There are many medications that will help your child manage aggression and destructiveness. The health care provider will discuss them with you.‖

c The parents are seeking a quick solution. Medications are generally not indicated for ODD. Comorbid conditions that increase defiant symptoms, such as ADHD, should be managed with medication, but no comorbid problem is identified in the question. The nurse should give information on helpful strategies to manage the adolescent's behavior.

An adolescent was arrested for prostitution and assault on a parent. The adolescent says, ―I hate my parents. They focus all attention on my brother, who's perfect in their eyes.‖ Which nursing diagnosis is most applicable? a. Disturbed personal identity related to acting out as evidenced by prostitution b. Hopelessness related to achievement of role identity as evidenced by feeling unloved by parents c. Defensive coping related to inappropriate methods of seeking parental attention as evidenced by acting out d. Impaired parenting related to inequitable feelings toward children as evidenced by showing preference for one child over another

c The patient demonstrates a failure to follow age-appropriate social norms and an inability to problem solve by using adaptive behaviors to meet life's demands and roles. The defining characteristics are not present for the other nursing diagnoses. The patient never mentioned hopelessness or disturbed personal identity. The problem relates to the patient's perceptions of parental behavior rather than the actual behavior.

A person has minor physical injuries after an auto accident. The person is unable to focus and says, "I feel like something awful is going to happen." This person has nausea, dizziness, tachycardia, and hyperventilation. What is the person's level of anxiety? a. Mild b. Moderate c. Severe d. Panic

c The person whose anxiety is severe is unable to solve problems and may have a poor grasp of what is happening in the environment. Somatic symptoms such as those described are usually present. The individual with mild anxiety is only mildly uncomfortable and may even find his or her performance enhanced. The individual with moderate anxiety grasps less information about a situation and has some difficulty with problem solving. The individual in panic will demonstrate markedly disturbed behavior and may lose touch with reality.

A cognitively impaired client has been a widow for 30 years. This client frantically tries to leave the facility, saying, "I have to go home to cook dinner before my husband arrives from work." To intervene with validation therapy, what should the nurse say? a. "You must come away from the door." b. "You have been a widow for many years." c. "You want to go home to prepare your husband's dinner?" d. "Your husband gets angry if you do not have dinner ready on time?"

c Validation therapy meets the client "where she or he is at the moment" and acknowledges the client's wishes. Validation does not seek to redirect, reorient, or probe. The distracters do not validate the client's feelings.

A new client beginning an alcohol rehabilitation program says, "I'm just a social drinker. I usually have one drink at lunch, two in the afternoon, wine at dinner, and a few drinks during the evening." Which responses by the nurse will be most therapeutic? (Select all that apply.) a. "I see," and use interested silence. b. "I think you are drinking more than you report." c. "Social drinkers have one or two drinks, once or twice a week." d. "You describe drinking steadily throughout the day and evening." e. "Your comments show denial of the seriousness of your problem."

c, d The correct answers give information, summarize, and validate what the client reported but are not strongly confrontational. Defenses cannot be removed until healthier coping strategies are in place. Strong confrontation does not usually take place so early in the program.

Which experiences are most likely to precipitate post-traumatic stress disorder (PTSD)? (Select all that apply.) a. A young adult bungee jumped from a bridge with a best friend. b. An 8-year-old child watched an R-rated movie with both parents. c. An adolescent was kidnapped and held for 2 years in the home of a sexual predator. d. A passenger was in a bus that overturned on a sharp curve and tumbled down an embankment. e. An adult was trapped for 3 hours at an angle in an elevator after a portion of the supporting cable breaks.

c, d, e PTSD usually occurs after a traumatic event that is outside the range of usual experience. Examples are childhood physical abuse, torture/kidnap, military combat, sexual assault, and natural disasters, such as floods, tornados, earthquakes, tsunamis; human disasters, such as a bus or elevator accident; or crime-related events, such being taken hostage. The common element in these experiences is the individual's extraordinary helplessness or powerlessness in the face of such stressors. Bungee jumps by adolescents are part of the developmental task and might be frightening, but in an exhilarating way rather than a harmful way. A child may be disturbed by an R-rated movie, but the presence of the parents would modify the experience in a positive way.

To plan effective care for clients diagnosed with somatic symptom disorders, the nurse should understand that clients have difficulty giving up the symptoms because of what fact? a. They are generally chronic. b. They have a physiological basis. c. They can be voluntarily controlled. d. They provide relief from health anxiety.

d At the unconscious level, the client's primary gain from the symptoms is anxiety relief. Considering that the symptoms actually make the client more psychologically comfortable and may also provide secondary gain, clients frequently fiercely cling to the symptoms. The symptoms tend to be chronic, but that does not explain why they are difficult to give up. The symptoms are not under voluntary control or physiologically based.

A 15-year-old ran away from home six times and was arrested for shoplifting. The parents told the Court, ―We can't manage our teenager.‖ The adolescent is physically abusive to the mother and defiant with the father. Which diagnosis is supported by this adolescent's behavior? a. Attention deficit hyperactivity disorder (ADHD) b. Posttraumatic stress disorder (PTSD) c. Intermittent explosive disorder d. CD

d CDs are manifested by a persistent pattern of behavior in which the rights of others and age- appropriate societal norms are violated. Intermittent explosive disorder is a pattern of behavioral outbursts characterized by an inability to control aggressive impulses in adults 18 years and older. Criteria for ADHD and PTSD are not met in the scenario.

For a client experiencing panic, which nursing intervention should be implemented first? a. Teach relaxation techniques. b. Administer an anxiolytic medication. c. Prepare to implement physical controls. d. Provide calm, brief, directive communication.

d Calm, brief, directive verbal interaction can help the client gain control of overwhelming feelings and impulses related to anxiety. Clients experiencing panic-level anxiety are unable to focus on reality; thus, learning relaxation techniques is virtually impossible. Administering anxiolytic medication should be considered if providing calm, brief, directive communication is ineffective. Although the client is disorganized, violence may not be imminent, ruling out the intervention of preparing for physical control until other less-restrictive measures are proven ineffective.

A nurse assesses a client with a tentative diagnosis of generalized anxiety disorder. Which question would be most appropriate for the nurse to ask? a. "Have you been a victim of a crime or seen someone badly injured or killed?" b. "Do you feel especially uncomfortable in social situations involving people?" c. "Do you repeatedly do certain things over and over again?" d. "Do you find it difficult to control your worrying?"

d Clients with generalized anxiety disorder frequently engage in excessive worrying. They are less likely to engage in ritualistic behavior, fear social situations, or have been involved in a highly traumatic event.

A client checks and rechecks electrical cords related to an obsessive thought that the house may burn down. The nurse and client explore the likelihood of an actual fire. The client states this event is not likely. This counseling demonstrates what appropriate principle of therapy? a. flooding. b. desensitization. c. relaxation technique. d. cognitive restructuring.

d Cognitive restructuring involves the client in testing automatic thoughts and drawing new conclusions. Desensitization involves graduated exposure to a feared object. Relaxation training teaches the client to produce the opposite of the stress response. Flooding exposes the client to a large amount of an undesirable stimulus in an effort to extinguish the anxiety response.

A person who feels unattractive repeatedly says, "Although I'm not beautiful, I am smart." This is an example of what defense mechanism? a. repression. b. devaluation. c. identification. d. compensation.

d Compensation is an unconscious process that allows us to make up for deficits in one area by excelling in another area to raise self-esteem. Repression unconsciously puts an idea, event, or feeling out of awareness. Identification is an unconscious mechanism calling for imitation of mannerisms or behaviors of another. Devaluation occurs when the individual attributes negative qualities to self or others.

A confused older adult client in a skilled nursing facility was asleep when unlicensed assistive personnel (UAP) entered the room quietly and touched the bed to see if it was wet. The client awakened and hit the UAP in the face. Which statement best explains the client's action? a. Older adult clients often demonstrate exaggerations of behaviors used earlier in life. b. Crowding in skilled nursing facilities increases an individual's tendency toward violence. c. The client learned violent behavior by watching other clients act out. d. The client interpreted the UAP's behavior as potentially harmful.

d Confused clients are not always able to evaluate the actions of others accurately. This client behaved as though provoked by the intrusive actions of the staff.

A client was arrested for breaking windows in the home of a former domestic partner. The client's history reveals childhood abuse by a punitive parent, torturing family pets, and an arrest for disorderly conduct. Which nursing diagnosis has priority? a. Risk for injury b. Ineffective coping c. Impaired social interaction d. Risk for other-directed violence

d Defining characteristics for risk for other-directed violence include a history of being abused as a child, having committed other violent acts, and demonstrating poor impulse control. There is no indicator that the client will experience injury. Ineffective coping and impaired social interaction have lower priorities.

A client undergoing diagnostic tests says, "Nothing is wrong with me except a stubborn chest cold." The spouse reports the client smokes, coughs daily, lost 15 pounds, and is easily fatigued. Which defense mechanism is the client using? a. Displacement b. Regression c. Projection d. Denial

d Denial is an unconscious blocking of threatening or painful information or feelings. Regression involves using behaviors appropriate at an earlier stage of psychosexual development. Displacement shifts feelings to a more neutral person or object. Projection attributes one's own unacceptable thoughts or feelings to another.

Which assessment findings are likely for an individual who recently injected heroin? a. Anxiety, restlessness, paranoid delusions b. Muscle aching, dilated pupils, tachycardia c. Heightened sexuality, insomnia, euphoria d. Drowsiness, constricted pupils, slurred speech

d Heroin, an opiate, is a central nervous system (CNS) depressant. Blood pressure, pulse, and respirations will be decreased, and attention will be impaired. The distracters describe behaviors consistent with amphetamine use, symptoms of narcotic withdrawal, and cocaine use.

A client with fears of serious heart disease was referred to the mental health center by a cardiologist after extensive diagnostic evaluation showed no physical illness. The client says, "My chest is tight, and my heart misses beats. I'm often absent from work. I don't go out much because I need to rest." Which health problem is most likely? a. Dysthymic disorder b. Somatic symptom disorder c. Antisocial personality disorder d. Illness anxiety disorder (hypochondriasis)

d Illness anxiety disorder (hypochondriasis) involves preoccupation with fears of having a serious disease even when evidence to the contrary is available. The preoccupation causes impairment in social or occupational functioning. Somatic symptom disorder involves fewer symptoms. Dysthymic disorder is a disorder of lowered mood. Antisocial disorder applies to a personality disorder in which the individual has little regard for the rights of others. See relationship to audience response question.

A student says, "Before taking a test, I feel very alert and a little restless." The nurse can correctly uses what term to document the student's experience? a. culturally influenced. b. displacement. c. trait anxiety. d. mild anxiety.

d Mild anxiety is rarely obstructive to the task at hand. It may be helpful to the client because it promotes study and increases awareness of the nuances of questions. The incorrect responses have different symptoms.

Which medication to maintain abstinence would most likely be prescribed for clients diagnosed with an addiction to either alcohol or opioids? a. Bromocriptine b. Methadone c. Disulfiram d. Naltrexone

d Naltrexone is useful for treating both opioid and alcohol addiction. An opioid antagonist blocks the action of opioids and the mechanism of reinforcement. It also reduces or eliminates alcohol craving. None of the other options are associated with such a response.

What is the correct etiology to complete this nursing diagnosis for a client diagnosed with dissociative identity disorder? a. obsessive fears of harming self or others. b. poor impulse control and lack of self-confidence. c. depressed mood secondary to nightmares and intrusive thoughts. d. cognitive distortions associated with unresolved childhood abuse issues.

d Nearly all clients with dissociative identity disorder have a history of childhood abuse or trauma. None of the other etiology statements is relevant.

Family members of an individual undergoing a residential alcohol rehabilitation program ask, "How can we help?" What is the nurse's best response? a. "Alcoholism is a lifelong disease. Relapses are expected." b. "Use search and destroy tactics to keep the home alcohol free." c. "It's important that you visit your family member on a regular basis." d. "Make your loved one responsible for the consequences of behavior."

d Often, the addicted individual has been enabled when others picked up the pieces for him or her. The individual never faced the consequences of his or her own behaviors, all of which relate to taking responsibility. Learning to face those consequences is part of the recovery process. The other options are codependent behaviors or are of no help.

A client is thin, tense, jittery, and has dilated pupils. The client says, "My heart is pounding in my chest. I need help." The client allows vital signs to be taken but then becomes suspicious and says, "You could be trying to kill me." The client refuses further examination. Abuse of which substance is most likely? a. phencyclidine (PCP) b. Heroin c. Barbiturates d. Amphetamines

d The physical symptoms are consistent with central nervous system (CNS) stimulation. Suspicion and paranoid ideation are also present. Amphetamine use is likely. PCP use would probably result in bizarre, violent behavior. Barbiturates and heroin would result in symptoms of CNS depression.

A soldier returned home last year after deployment to a war zone. The soldier's spouse complains, "We were going to start a family, but now he won't talk about it. He will not look at children. I wonder if we're going to make it as a couple." Select the nurse's best response. a. "Posttraumatic stress disorder (PTSD) often changes a person's sexual functioning." b. "I encourage you to continue to participate in social activities where children are present." c. "Have you talked with your spouse about these reactions? Sometimes we just need to confront behavior." d. "Posttraumatic stress disorder often strains relationships. Here are some community resources for help and support."

d PTSD precipitates changes that can lead to divorce. It is important to provide support to both the veteran and spouse. Confrontation will not be effective. While it is important to provide information, on-going support will be more effective.

A person who has been unable to leave home for more than a week because of severe anxiety says, "I know it does not make sense, but I just can't bring myself to leave my apartment alone." Which nursing intervention is appropriate? a. Help the person use online video calls to provide interaction with others. b. Advise the person to accept the situation and use a companion. c. Ask the person to explain why the fear is so disabling. d. Teach the person to use positive self-talk techniques.

d Positive self-talk, a form of cognitive restructuring, replaces negative thoughts such as "I can't leave my apartment" with positive thoughts such as "I can control my anxiety." This technique helps the client gain mastery over the symptoms. The other options reinforce the sick role.

A client diagnosed with an antisocial personality disorder was treated several times for substance abuse, but each time the client relapsed. Which treatment approach is most appropriate? a. 1-week detoxification program b. Long-term outpatient therapy c. 12-step self-help program d. Residential program

d Residential programs and therapeutic communities help clients change lifestyles, abstain from drugs, eliminate criminal behaviors, develop employment skills, become self-reliant, and practice honesty. Residential programs are more effective for clients with antisocial tendencies than outpatient programs.

A child drowned while swimming in a local lake 2 years ago. Which behavior indicates the child's parents have adapted to their loss? a. visiting their child's grave daily. b. maintaining their child's room as the child left it 2 years ago. c. keeping a place set for the dead child at the family dinner table. d. throwing flowers on the lake at each anniversary date of the accident.

d Resilience refers to positive adaptation or the ability to maintain or regain mental health despite adversity. Loss of a child is among the highest risk situations for an adjustment disorder and maladaptive grieving. The parents who throw flowers on the lake on each anniversary date of the accident are openly expressing their feelings. The other behaviors are maladaptive because of isolating themselves and/or denying their feelings. After 2 years, the frequency of visiting the grave should have decreased.

Four teenagers died in an automobile accident. One week later, which behavior by the parents of these teenagers most clearly demonstrates resilience? a. visiting their teenager's grave daily. b. returning immediately to employment. c. discussing the accident within the family only. d. creating a scholarship fund at their child's high school.

d Resilience refers to positive adaptation or the ability to maintain or regain mental health despite adversity. Loss of a child is among the highest risk situations for maladaptive grieving. The parents who create a scholarship fund are openly expressing their feelings and memorializing their child. The other parents in this question are isolating themselves and/or denying their feelings. Visiting the grave daily shows active continued mourning but is not as strongly indicative of resilience as the correct response.

The family of a child diagnosed with an impulse control disorder needs help to function more adaptively. Which aspect of the child's plan of care will be provided by an advanced practice nurse rather than a staff nurse? a. Leading an activity group b. Providing positive feedback c. Formulating nursing diagnoses d. Dialectical behavioral therapy (DBT)

d The advanced practice nurse role includes individual, group, and family psychotherapist; educator of nurses, other professions, and the community; clinical supervisor; consultant to professional and nonprofessional groups; and researcher. DBT is an aspect of psychotherapy. The distracters describe actions of a nurse generalist.

A 12-year-old has engaged in bullying for several years. The parents say, ―We can't believe anything our child says.‖ Recently this child shot a dog with a pellet gun and set fire to a neighbor's trash bin. The child's behaviors support the diagnosis of a. ADHD. b. intermittent explosive disorder. c. oppositional defiant disorder (ODD). d. CD.

d The behaviors mentioned are most consistent with criteria for CD, including aggression against people and animals; destruction of property; deceitfulness; rule violations; and impairment in social, academic, or occupational functioning. Intermittent explosive disorder is a pattern of behavioral outbursts characterized by an inability to control aggressive impulses in adults 18 years and older. The behaviors are not consistent with attention deficit or oppositional defiant disorder (ODD).

A nurse assessing a client diagnosed with a somatic symptom disorder is most likely to note which client characteristic? a. sees a relationship between symptoms and interpersonal conflicts. b. has little difficulty communicating emotional needs to others. c. rarely derives personal benefit from the symptoms. d. has altered comfort and activity needs.

d The client frequently has altered comfort and activity needs associated with the symptoms displayed (fatigue, insomnia, weakness, tension, pain, etc.). In addition, hygiene, safety, and security needs may also be compromised. The client is rarely able to see a relation between symptoms and events in his or her life, which is readily discernible to health professionals. Clients with somatic symptom disorders often derive secondary gain from their symptoms and/or have considerable difficulty identifying feelings and conveying emotional needs to others.

An adolescent diagnosed with an impulse control disorder says, ―I want to die. I spend my time getting even with people who hurt me.‖ When asked about a suicide plan, the adolescent replies, ―I'll jump from a bridge near my home. My father threw kittens off that bridge and they died. Rate the suicide risk. a. Absent b. Low c. Moderate d. High

d The suicide risk is high. The child is experiencing feelings of hopelessness and helplessness. The method described is lethal, and the means to carry out the plan are available.

A store clerk was killed during a robbery 2 weeks ago. His widow, who has a long history of schizoaffective disorder, cries spontaneously when talking about his death. What is the nurse's most therapeutic response? a. "Are you taking your medications the way they are prescribed?" b. "This loss is harder to accept because of your mental illness. Do you think you should be hospitalized?" c. "I'm worried about how much you are crying. Your grief over your husband's death has gone on too long." d. "The unexpected death of your husband is very painful. I'm glad you are able to talk about your feelings."

d The client is expressing feelings related to the loss, and this is an expected and healthy behavior. This client is at risk for a maladaptive response because of the history of a serious mental illness, but the nurse's priority intervention is to form a therapeutic alliance and support the client's expression of feelings. Crying at 2 weeks after his death is expected and normal.

A client admitted yesterday for injuries sustained while intoxicated believes insects are crawling on the bed. The client is anxious, agitated, and diaphoretic. What is the priority nursing diagnosis? a. Disturbed sensory perception b. Ineffective coping c. Ineffective denial d. Risk for injury

d The client's clouded sensorium, sensory perceptual distortions, and poor judgment predispose a risk for injury. Safety is the nurse's priority. The other diagnoses may apply but are not the priorities of care since none are related to the client's physical needs.

An emergency code was called after a client pulled a dinner knife from a pocket and threatened, "I will kill anyone who tries to get near me." The client was safely disarmed and placed in seclusion. What is the justification for this use of seclusion? a. The client was threatening to others. b. The client was experiencing psychosis. c. The client presented an undeniable escape risk. d. The client presented a clear and present danger to others.

d The client's threat to kill self or others with the knife he possessed constituted a clear and present danger to self and others. The distracters are not sufficient reasons for seclusion.

A client diagnosed with alcohol use disorder says, "Drinking helps me cope with being a single parent." Which therapeutic response by the nurse would help the client conceptualize the drinking objectively? a. "Sooner or later, alcohol will kill you. Then what will happen to your children?" b. "I hear a lot of defensiveness in your voice. Do you really believe this?" c. "If you were coping so well, why were you hospitalized again?" d. "Tell me what happened the last time you drank."

d The correct response will help the client see alcohol as a cause of the problems, not a solution, and begin to take responsibility. This approach can help the client become receptive to the possibility of change. The other responses directly confront and attack defenses against anxiety that the client still needs. They reflect the nurse's frustration with the client.

A client sat in silence for 20 minutes after a therapy appointment, appearing tense and vigilant. The client abruptly stood, paced back and forth, clenched and unclenched fists, and then stopped and stared in the face of a staff member. What is the client likely doing? a. demonstrating withdrawal. b. working though angry feelings. c. attempting to use relaxation strategies. d. exhibiting clues to potential aggression.

d The description of the client's behavior shows the classic signs of someone whose potential for aggression is increasing. The scenario does not support any of the other options.

A soldier returned 3 months ago from a combat zone and was diagnosed with post-traumatic stress disorder (PTSD). Which social event would be most disturbing for this soldier? a. Halloween festival with neighborhood children b. Singing carols around a Christmas tree c. A family outing to the seashore d. Fireworks display on July 4th

d The exploding noises associated with fireworks are likely to provoke exaggerated responses for this soldier. The distracters are not associated with offensive sounds.

Which goal for treatment of alcohol use disorder should the nurse address first? a. Learn about addiction and recovery. b. Develop alternate coping strategies. c. Develop a peer support system. d. Achieve physiological stability.

d The individual must have completed withdrawal and achieved physiological stability before he or she is able to address any of the other treatment goals.

What is an essential difference between somatic symptom disorders and factitious disorders? a. Somatic symptom disorders are under voluntary control, whereas factitious disorders are unconscious and automatic. b. Factitious disorders are precipitated by psychological factors, whereas somatic symptom disorders are related to stress. c. Factitious disorders are individually determined and related to childhood sexual abuse, whereas somatic symptom disorders are culture bound.d. d. Factitious disorders are under voluntary control, whereas somatic symptom disorders involve expression of psychological stress through somatization.

d The key is the only fully accurate statement. Somatic symptom disorders involve expression of stress through bodily symptoms and are not under voluntary control or culture bound. Factitious disorders are under voluntary control. See relationship to audience response question.

A client with severe burn injuries is irritable, angry, and belittles the nurses. As a nurse changes a dressing, the client screams, "Don't touch me! You are so stupid. You will make it worse!" Which action by the nurse will best help to diffuse the client's anger? a. Stop the dressing change and say, "I will leave the supplies so that you can change your own dressing." b. Continue the dressing change and say, "This dressing change is necessary because you were careless with fire." c. Discontinue the dressing change, tell the client, "I will return when you gain control of yourself," and leave the room. d. Continue the dressing change and say, "Dressing changes are needed to prevent infection. What are your ideas about how to make it less painful?"

d The nurse should not respond personally to the client's comments. The correct answer objectively gives the client information that may lead to lowering his anger and engages the client in problem solving. The incorrect options will escalate the client's anger by belittling or escalating the client's sense of powerlessness. Dressing changes are needed for the client's physiological integrity; therefore, the nurse should not abandon the responsibility to perform them.

Which history information from a client's record would indicate marginal coping skills and the need for careful assessment of the risk for violence? a. academic problems. b. family involvement. c. childhood trauma. d. substance abuse.

d The nurse should suspect marginal coping skills in a client with substance abuse. They are often anxious, may be concerned about inadequate pain relief, and may have personality styles that externalize blame. The incorrect options do not signal as high a degree of risk as substance abuse.


Kaugnay na mga set ng pag-aaral

IB Cell Cycle, DNA, and DNA Replication

View Set

2nde BAC PRO HELLO Introduce yourself

View Set

(1) Completing the Application, Underwriting, and Delivering Policy

View Set

ECN CH.15 Aggregate Demand and Aggregate Supply Quiz

View Set